Buy BOOKS at Discounted Price

Determinants

Class 12th Mathematics RD Sharma Volume 1 Solution
Exercise 6.1
  1. a = [ll 5&20 0&-1] Write the minors and cofactors of each element of the first…
  2. a = [cc -1&4 2&3] Write the minors and cofactors of each element of the first…
  3. a = [ccc 1&-3&2 4&-1&2 3&5&2] Write the minors and cofactors of each element of…
  4. a = [lll 1 1 1] Write the minors and cofactors of each element of the first…
  5. a = [lll 0&2&6 1&5&0 3&7&1] Write the minors and cofactors of each element of…
  6. a = [lll a h g] Write the minors and cofactors of each element of the first…
  7. a = [rrrr 2&-1&0&1 -3&0&1&-2 1&1&-1&1 2&-1&5&0] Write the minors and cofactors…
  8. Evaluate the following determinants: i. | cc x&-7 x&5x+1 | ii. | cc costheta…
  9. Evaluate | ccc 2&3&7 13&17&5 15&20&12 |^2
  10. Show that | cc sin10^circle & - cos10^circle sin80^circle & cos80^circle | = 1…
  11. Evaluate | rrr 2&3&-5 7&1&-2 -3&4&1 | by two methods.
  12. Evaluate : delta = | ccc 0 &-cosalpha -sinalpha &0 cosalpha &-sinbeta &0 |…
  13. Evaluate : delta = | ccc cosalpha cosbeta sinbeta &-sinalpha -sinbeta &0…
  14. If a = [ll 2&5 2&1] and b = [rr 4&-3 2&5] , verify that |AB| = |A| |B|.…
  15. If a = [lll 1&0&1 0&1&2 0&0&4] , then show that |3a| = 27|a| .
  16. | ll 2&4 5&1 | = | cc 2x&4 6 | Find the value of x, if
  17. | ll 2&3 4&5 | = | ll x&3 2x&5 | Find the value of x, if
  18. | ll 3 x&1 | = | ll 3&2 4&1 | Find the value of x, if
  19. | rr 3x&7 2&4 | = 10 Find the value of x, if
  20. | rr x+1 x-3+2 | = | rr 4&-1 1&3 | Find the value of x, if
  21. | rr 2x&5 8 | = | ll 6&5 8&3 | Find the value of x, if
  22. Find the integral value of x, if | lll x^2 &1 0&2&1 3&1&4 | = 28
  23. a = [ll 1+x&7 3-x&8] For what value of x matrix A is singular?
  24. a = [ccc x-1&1&1 1&1 1&1] For what value of x matrix A is singular?…
Exercise 6.2
  1. | ccc 1&3&5 2&6&10 31&11&38 | Evaluate the following determinant:…
  2. | ccc 67&19&21 39&13&14 81&24&26 | Evaluate the following determinant:…
  3. | lll a h g | Evaluate the following determinant:
  4. | rr 1&-3&2 4&-1&2 3&5&2 | Evaluate the following determinant:
  5. | ccc 1&4&9 4&9&16 9&16&25 | Evaluate the following determinant:
  6. | rrr 6&-3&2 2&-1&2 -10&5&2 | Evaluate the following determinant:…
  7. | cccc 1&3&9&27 3&9&27&1 9&27&1&3 27&1&3&9 | Evaluate the following…
  8. | rrr 102&18&36 1&3&4 17&3&6 | Evaluate the following determinant:…
  9. | ccc 8&2&7 12&3&5 16&4&3 | Without expanding, show that the value of each of…
  10. | rrr 6&-3&2 2&-1&2 -10&5&2 | Without expanding, show that the value of each of…
  11. | ccc 2&3&7 13&17&5 15&20&12 | Without expanding, show that the value of each…
  12. | lll 1/a& a^2 1/b& b^2 1/c& c^2 | Without expanding, show that the value of…
  13. Without expanding, show that the value of each of the following determinants is…
  14. | 1aa^2 - bc 1bb^2 - ac 1cc^2 - ab | Without expanding, show that the value of…
  15. | ccc 49&1&6 39&7&4 26&2&3 | Without expanding, show that the value of each of…
  16. | rrr 0 -x&0 -y&-z&0 | Without expanding, show that the value of each of the…
  17. | ccc 1&43&6 7&35&4 3&17&2 | Without expanding, show that the value of each of…
  18. | cccc 1^2 & 2^2 & 3^2 & 4^2 2^2 & 3^2 & 4^2 & 5^2 3^2 & 4^2 & 5^2 & 6^2 4^2 &…
  19. | ccc a a+2x+2y+2z x | Without expanding, show that the value of each of the…
  20. | ll (2^x + 2^-x)^2 & (2^x - 2^-x)^2 &1 (3^x + 3^-x)^2 & (3^x + 3^-x)^2 &1 (4^x…
  21. | lll sinalpha & cos (alpha + delta) sinbeta & cos (beta + delta) singamma &…
  22. | ccc sin^223^circle & sin^267^circle & cos180^circle - sin^267^circle & -…
  23. | ccc cos (x+y) & - sin (x+y) sinx -cosx | Without expanding, show that the…
  24. | ccc root 23 + root 3 & root 5 & root 5 root 15 + root 46 &5& root 10 3 + root…
  25. | ccc sin^2a&1 sin^2b&1 sin^2c | , where A, B, C are the angles of ΔABC.…
  26. | lll a+c& a^2 b+a& b^2 c+b& c^2 | Evaluate the following:
  27. | lll 1 1 1 | Evaluate the following:
  28. | ccc x + lambda x& x + lambda x x& x + lambda | Evaluate the following:…
  29. | lll a c b | Evaluate the following:
  30. | ll x&1&1 1&1 1&1 | Evaluate the following:
  31. | ccc 0& xy^2 & xz^2 x^2y&0& yz^2 x^2z& zy^2 &0 | Evaluate the following:…
  32. | ccc 1+x x+y x+z | Evaluate the following:
  33. If delta = | lll 1& x^2 1& y^2 1& z^2 | , delta_1 = | lll 1&1&1 yz x | , then…
  34. | ccc a a-b b+cc+a+b | = a^3 + b^3 + c^3 - 3abc Prove the following identities:…
  35. | ll b+ca-b c+ab-c a+bc-a | = 3abc-a^3 - b^3 - c^3 Prove the following…
  36. | a+bb+cc+a b+cc+aa+b c+a+bb+c | = 2 | ccc a b c | Prove the following…
  37. | ccc a+b+2c c+c+2a c+a+2b | = 2 (a+b+c)^3 Prove the following identities:…
  38. | rrr a-b-c&2a&2a 2b&2b 2c&2cc-a-b | = (a+b+c)^3 Prove the following…
  39. | lll 1+c& b^2 + c^2 1+a& c^2 + a^2 1+b& a^2 + b^2 | = (a-b) (b-c) (c-a) Prove…
  40. | ccc a+b+2b a+2b+b a+b+2b | = 9 (a+b) b^2 Prove the following identities:…
  41. | lll 1 1 1 | = | lll 1& a^2 1& b^2 1& c^2 | Prove the following identities:…
  42. | ccc z z^2 & x^2 & y^2 z^4 & x^4 & y^4 | = | ccc x x^2 & y^2 & z^2 x^4 & y^4 &…
  43. | lll (b+c)^2 & b^2 (c+a)^2 & b^2 (a+b)^2 & c^2 | = (a-b) (b-c) (c-a) (a+b)…
  44. | lll (b+1) (a+2) +2&1 (a+2) (a+3) +3&1 (a+3) (a+4) +4&1 | = - 2 Prove the…
  45. | lll a^2 & a^2 - (b-c)^2 b^2 & b^2 - (c-a)^2 c^2 & c^2 - (a-b)^2 | = (a-b)…
  46. | ccc 1& a^2 + bc & a^3 1& b^2 + ca & b^3 1& c^2 + ab & c^3 | = - (a-b) (b-c)…
  47. | ccc a^2 & ac+c^2 a^2 + ab & b^2 ab& b^2 + bc & c^2 | = 4a^2b^2c^2 Prove the…
  48. | ccc x+4 x+4 x+4 | = 16 (3x+4) Prove the following identities:
  49. | ccc 1&1+p&1+p+q 2&3+2p&4+3p+2q 3&3+3p&10+6p+3q | = 1 Prove the following…
  50. | ccc a a-c a-b | = (a + b - c)(b + c - a)(c + a - b) Prove the following…
  51. | ccc a^2 &2ab& b^2 b^2 & a^2 &2ab 2ab& b^2 & a^2 | = (a^3 + b^3)^2 Prove the…
  52. | lll 1& a^2 a^2 &1 a& a^2 &1 | = (a^3 - 1)^2 Prove the following identities -…
  53. | ccc a+b+c&-c&-b -c+b+c&-c -b&-a+b+c | = 2(a + b)(b + c)(c + a) Prove the…
  54. | ccc b+c b+a c+b | = 4abc Prove the following identities -
  55. | ccc b^2 + c^2 ba& c^2 + a^2 ca& a^2 + b^2 | = 4a^2b^2c^2 Prove the following…
  56. | ccc 0^2a^2a a^2b&0^2b a^2c^2c&0 | = 2a^3b^3c^3 Prove the following identities…
  57. | ccc a^2 + b^2/c a& b^2 + c^2/a b& c^2 + a^2/b | = 4abc Prove the following…
  58. | cc -bc& b^2 + bc & c^2 + bc a^2 + ac &-ac& c^2 + ac a^2 + ab & b^2 + ab &-ab…
  59. | ccc x + lambda &2x&2x 2x& x + lambda &2x 2x&2x& x + lambda | = (5x + lambda)…
  60. | ccc x+4&2x&2x 2x+4&2x 2x&2x+4 | = (5x+4) (4-x)^2 Prove the following…
  61. | ccc y+z z+x y+y | = 4xyz Prove the following identities -
  62. | ccc - a (b^2 + c^2 - a^2) & 2b^3 & 2c^3 2a^3 & - b (c^2 + a^2 - b^2) 2c^3…
  63. | ccc 1+a&1&1 1&1+a&1 1&1&1+a | = a^3 + 3a^2 Prove the following identities -…
  64. | ccc 2y&2y 2z&2z x-y-z&2x&2x | = (x+y+z)^3 Prove the following identities -…
  65. Prove the following identities -
  66. | ccc a+x x+y x+z | = a^2 (a+x+y+z) Prove the following identities -…
  67. | lll a^3 &2 b^3 &2 c^3 &2 | = 2(a - b)(b - c)(c - a)(a + b + c) Prove the…
  68. Without expanding, prove that .
  69. Show that | x+1x+2x+a x+2x+3x+b x+3x+4x+c | = 0 where a,b,c are in A.P.…
  70. Show that | x-3x-4x-a x-2x-3x - beta x-1x-2x-y | = 0 where α, β and γ are in…
  71. If a, b, c are real numbers such that | lll b+c+a+b c+a+b+c a+b+c+a | = 0 ,…
  72. If | lll p a a | = 0 , find the value of p/p-a + q/q-b + r/r-c , p not equal a…
  73. Show that x = 2 is a root of the equation | rrr x&-6&-1 2&-3x -3&2x+2 | = 0 and…
  74. | ccc x+a a+b a+c | = 0 Solve the following determinant equations:…
  75. | ccc x+a x+a x+a | = 0 , a not equal 0 Solve the following determinant…
  76. | ccc 3x-8&3&3 3&3x-8&3 3&3&3x-8 | = 0 Solve the following determinant…
  77. | lll 1& x^2 1& a^2 1& b^2 | = 0 , a not equal b Solve the following…
  78. | ccc x+1&3&5 2+2&5 2&3+4 | = 0 Solve the following determinant equations:…
  79. | lll 1& x^3 1& b^3 1& c^3 | = 0 , b not equal c Solve the following…
  80. | ccc 15-2x&11-3x&7-x 11&17&14 10&16&13 | = 0 Solve the following determinant…
  81. | ccc 1&1 p+1+1+x 3+1+2 | = 0 Solve the following determinant equations:…
  82. | ccc 3&-2 -7&8 -11&14&2 | = 0 Solve the following determinant equations:…
  83. If a, b and c are all non-zero and | ccc 1+a&1&1 1&1+b&1 1&1&1+c | = 0 , then…
  84. If | ccc a a-x a-x | = 0 , then using properties of determinants, find the…
Exercise 6.3
  1. (3, 8), (- 4, 2) and (5, - 1) Find the area of the triangle with vertices at…
  2. (2, 7) (1, 1) and (10, 8) Find the area of the triangle with vertices at the…
  3. (- 1, - 8), (- 2, - 3) and (3, 2) Find the area of the triangle with vertices…
  4. (0, 0) (6, 0) and (4, 3) Find the area of the triangle with vertices at the…
  5. (1, - 1), (2, 1) and (4, 5) Using determinants show that the following points…
  6. (3, - 2), (8, 8) and (5, 2) Using determinants show that the following points…
  7. (2, 3), (- 1, - 2) and (5, 8) Using determinants show that the following points…
  8. If the points (a, 0), (o, b) and (1, 1) are collinear, prove that a + b = ab.…
  9. Using determinants prove that the points (a, b) (a’, b’) and (a - a’, b - b’)…
  10. Find the value of λ so that the points (1, - 5), (- 4, 5) and (λ, 7) are…
  11. Find the value of x if the area of a triangle is 35 square cms with vertices (x,…
  12. Using determinants, find the area of a triangle whose vertices are (1, 4), (2,…
  13. Using determinants, find the area of the triangle with vertices (- 3, 5), (3, -…
  14. Using determinants, find the value of k so that the points (k, 2 - 2 k), (- k +…
  15. If the points (x, 2), (5, - 2) and (8, 8) are collinear, find x using…
  16. If the points (3, - 2), (x,2) and (8,8) are collinear, find x using…
  17. (1, 2) and (3, 6) Using determinants, find the equation of the line joining…
  18. (3, 1) and (9, 3) Using determinants, find the equation of the line joining…
  19. (k,0), (4,0) and (0,2) Find values of K, if the area of a triangle is 4 square…
  20. (- 2,0), (0, 4) and (0, k) Find values of K, if the area of a triangle is 4…
Exercise 6.4
  1. x - 2y = 4 - 3x + 5y = - 7 Solve the following systems of linear equations by…
  2. 2x - y = 1 7x - 2y = - 7 Solve the following systems of linear equations by…
  3. 2x - y = 17 3x + 5y = 6 Solve the following systems of linear equations by…
  4. 3x + y = 19 3x - y = 23 Solve the following systems of linear equations by…
  5. 2x - y = - 2 3x + 4y = 3 Solve the following systems of linear equations by…
  6. 3x + ay = 4 2x + ay = 2, a≠0 Solve the following systems of linear equations by…
  7. 2x + 3y = 10 x + 6y = 4 Solve the following systems of linear equations by…
  8. 5x + 7y = - 2 4x + 6y = - 3 Solve the following systems of linear equations by…
  9. 9x + 5y = 10 3y - 2x = 8 Solve the following systems of linear equations by…
  10. x + 2y = 1 3x + y = 4 Solve the following systems of linear equations by…
  11. 3x + y + z = 2 2x - 4y + 3z = - 1 4x + y - 3z = - 11 Solve the following system…
  12. x - 4y - z = 11 2x - 5y + 2z = 39 - 3x + 2y + z = 1 Solve the following system…
  13. 6x + y - 3z = 5 X + 3y - 2z = 5 2x + y + 4z = 8 Solve the following system of…
  14. x + y = 5 y + z = 3 x + z = 4 Solve the following system of the linear…
  15. 2y - 3z = 0 X + 3y = - 4 3x + 4y = 3 Solve the following system of the linear…
  16. 5x - 7y + z = 11 6x - 8y - z = 15 3x + 2y - 6z = 7 Solve the following system…
  17. 2x - 3y - 4z = 29 - 2x + 5y - z = - 15 3x - y + 5z = - 11 Solve the following…
  18. x + y = 1 x + z = - 6 x - y - 2z = 3 Solve the following system of the linear…
  19. x + y + z + 1 = 0 ax + by + cz + d = 0 a^2 x + b^2 y + c^2 z + d^2 = 0 Solve…
  20. x + y + z + w = 2 x - 2y + 2z + 2w = - 6 2x + y - 2z + 2w = - 5 3x - y + 3z -…
  21. 2x - 3z + w = 1 x - y + 2w = 1 - 3y + z + w = 1 x + y + z = 1 Solve the…
  22. 2x - y = 5 4x - 2y = 7 Show that each of the following systems of linear…
  23. 3x + y = 5 - 6x - 2y = 9 Show that each of the following systems of linear…
  24. 3x - y + 2z = 3 2x + y + 3z = 5 x - 2y - z = 1 Show that each of the following…
  25. x + y + z = 3 2x - y + z = 2 3x + 6y + 5z = 20. Show that each of the following…
  26. x - y + z = 3 2x + y - z = 2 - x - 2y + 2z = 1 Show that each of the following…
  27. x + 2y = 5 3x + 6y = 15 Show that each of the following systems of linear…
  28. x + y - z = 0 x - 2y + z = 0 3x + 6y - 5z = 0 Show that each of the following…
  29. 2x + y - 2z = 4 x - 2y + z = - 2 5x - 5y + z = - 2 Show that each of the…
  30. x - y + 3z = 6 x + 3y - 3z = - 4 5x + 3y + 3z = 10 Show that each of the…
  31. A salesman has the following record of sales during three months for three…
  32. An automobile company uses three types of steel S1, S2 and S3 for producing…
Exercise 6.5
  1. x + y - 2z = 0 2x + y - 3z = 0 5x + 4y - 9z = 0 Solve each of the following…
  2. 2x + 3y + 4z = 0 X + y + z = 0 2x + 5y - 2z = 0 Solve each of the following…
  3. 3x + y + z = 0 x - 4y3z = 0 2x + 5y - 2z = 0 Solve each of the following systems…
  4. Find the real values of λ for which the followings system of linear equations…
  5. If a,b,c are non - zero real numbers and if the system of equations (a - 1) x =…
Mcq
  1. If A and B are square matrices or order 2, then det (A + B) = 0 is possible only when…
  2. Which of the following is not correct? Mark the correct alternative in the following:…
  3. If a = | {lll} {a_{11}}&{a_{12}}&{a_{13}} {a_{21}}&{a_{22}}&{a_{23}}…
  4. Which of the following is not correct in a given determinant of A, where A = [aij]3 ×…
  5. Let | {lll} {x}&{2}&{x} { x^{2} } &{x}&{6} {x}&{x}&{6} | = ax^{4} + bx^{3} + cx^{2} +…
  6. The value of the determinant is independent of Mark the correct alternative in the…
  7. If , then Mark the correct alternative in the following:
  8. If d_{k} = | {ccc} {1}&{n}&{n} {2k}& { n^{2} + n+2 } & { n^{2} + n } {2k-1}& { n^{2} }…
  9. Let | {ccc} { x^{2} + 3x } &{x-1}&{x+3} {x+1}&{-2x}&{x-4} {x-3}&{x+4}&{3x} | = ax^{4}…
  10. Using the factor theorem it is found that a + b, b + c and c + a are three factors of…
  11. If a, b, c are distinct then the value of x satisfying | {ccc} {0}& { x^{2} - a } & {…
  12. If the determinant | {ccc} {a}&{b}& { 2a alpha +3b } {b}&{c}& { 2b alpha+3c } { 2a…
  13. If ω is a non-real cube root of unity and n is not a multiple of 3, then is equal to…
  14. If , then the value of sum _ { r = 1 } ^{n}a_{r} is Mark the correct alternative in…
  15. If a 0 and discriminant of ax2 + 2bx + c is negative, then delta = | {ccc}…
  16. The value of | {ccc} { 5^{2} } & { 5^{3} } & { 5^{4} } { 5^{3} } & { 5^{4} } & {…
  17. | {cc} {log_{3}512}&{log_{4}3} {log_{3}8}&{log_{4}9} | x | {ll} {log_{2}3}&{log_{8}3}…
  18. If a, b, c are in A.P., then the determinant | {lll} {x+2}&{x+3}&{x+2a}…
  19. If A + B + C = π, then the value of | {ccc} { sin (a+b+c) } & { sin (a+c) } &{cosc}…
  20. The number of distinct real roots of lies in the interval - { pi }/{4} less than…
  21. Let a = [ {ccc} {1}&{sintegrate heta }&{1} {-sintheta }&{1}&{sintheta}…
  22. If | {cc} {2x}&{5} {8}&{x} | = | {cc} {6}&{-2} {7}&{3} | , then x = Mark the correct…
  23. If f (x) = | {ccc} {0}&{x-a}&{x-b} {x+a}&{0}&{x-c} {x+b}&{x+c}&{0} | , then Mark the…
  24. The value of the determinant | {ccc} {a-b}&{b+c}&{a} {b-a}&{c+a}&{b} {c-a}&{a+b}&{c}…
  25. If x, y, z are different from zero and | {ccc} {1+x}&{1}&{1} {1}&{1+y}&{1}…
  26. The determinant | {lll} { b^{2} - ab } &{b-c}&{bc-ac} { ab-a^{2} } &{a-b}& { a^{2} -…
  27. If x, y ∈ R, then the determinant delta = | {ccc} {cosx}&{-sinx}&{1}…
  28. The maximum value of delta = | {ccc} {1}&{1}&{1} {1}&{1+sintegrate heta }&{1}…
  29. The value of the determinant | {ccc} {x}&{x+y}&{x+2y} {x+2y}&{x}&{x+y}…
  30. Let f (x) = | {ccc} {cosx}&{x}&{1} {2sinx}&{x}&{2x} {sinx}&{x}&{x} | , then lim_ { x…
  31. There are two values of a which makes the determinant delta = | {ccc} {1}&{-2}&{5}…
  32. If | {lll} {a}&{p}&{x} {b}&{q}&{y} {c}&{r}&{z} | = 16 , t hen the value of | {lll}…
  33. The value of is Mark the correct alternative in the following:
Very Short Answer
  1. If A is a singular matrix, then write the value of |A|.
  2. Write the value of the determinant | {ccc} {2}&{3}&{4} {2x}&{3x}&{4x} {5}&{6}&{8} | .…
  3. State whether the matrix [ {ll} {2}&{3} {6}&{4} ] is singular or non-singular.…
  4. Find the value of the determinant | {cc} {4200}&{4201} {4202}&{4203} | .…
  5. Find the value of the determinant | {lll} {101}&{102}&{103} {104}&{105}&{106}…
  6. Write the value of the determinant | {lll} {a}&{1}&{b+c} {b}&{1}&{c+a} {c}&{a}&{a+b} |…
  7. If a = [ {ll} {0}&{i} {i}&{1} ] and b = [ {ll} {0}&{1} {1}&{1} ] , find the value of…
  8. If a = [ {cc} {1}&{2} {3}&{-1} ] and b = [ {cc} {1}&{0} {-1}&{0} ] , find |AB|.…
  9. Evaluate: | {cc} {4785}&{4787} {4789}&{4791} | .
  10. If w is an imaginary cube root of unity, find the value of | {ccc} {1}&{w}& { w^{2} }…
  11. If a = [ {cc} {1}&{2} {3}&{-1} ] and b = [ {ll} {1}&{-4} {3}&{-2} ] , find |ab| .…
  12. If A = [aij] is a 3 × 3 diagonal matrix such that a11 = 1, a22 = 2 and a33 = 3, then…
  13. If A = [aij] is a 3 × 3 scalar matrix such that a11 = 2, then write the value of |A|.…
  14. If I3 denotes identity matrix of order 3 × 3, write the value of its determinant.…
  15. A matrix A of order 3 × 3 has determinant 5. What is the value of |3A|?…
  16. On expanding by first row, the value of the determinant of 3 × 3 square matrix A =…
  17. Let A = [aij] be a square matrix of order 3 × 3 and Cij denote cofactor of aij in A.…
  18. In question 18, write the value of a11C21 + a12C22 + a13C23.
  19. Write the value of | {cc} { sin20^{degree } } & { - cos20^{circ} } { sin70^{circ} }…
  20. If A is a square matrix satisfying ATA = I, write the value of |A|.…
  21. If A and B are square matrices of the same order such that |A| = 3 and AB = I, then…
  22. A is skew-symmetric of order 3, write the value of |A|.
  23. If A is a square matrix of order 3 with determinant 4, then write the value of |-A|.…
  24. If A is a square matrix such that |A| = 2, write the value of |AAT|.…
  25. Find the value of the determinant | {ccc} {243}&{156}&{300} {81}&{52}&{100}…
  26. Write the value of the determinant | {rrr} {2}&{-3}&{5} {4}&{-6}&{10} {6}&{-9}&{15} |…
  27. If the matrix [ {cc} {5x}&{2} {-10}&{1} ] is singular, find the value of x.…
  28. If A is a square matrix of order n × n such that |A| = λ, then write the value of…
  29. Find the value of the determinant | {ccc} { 2^{2} } & { 2^{3} } & { 2^{4} } { 2^{3} }…
  30. If A and B are non-singular matrices of the same order, write whether AB is singular…
  31. A matrix of order 3 × 3 has determinant 2. What is the value of |A(3I)|, where I is…
  32. If A and B are square matrices of order 3 such that |A| = -1, |B| = 3, then find the…
  33. Write the value of | {cc} {a+ib}&{c+id} {-c+id}&{a-ib} | .
  34. Write the cofactor of a12 in the matrix [ {ccc} {2}&{-3}&{5} {6}&{0}&{4} {1}&{5}&{-7}…
  35. If [ {ll} {2x+5}&{3} {5x+2}&{9} ] = 0 find x.
  36. Find the value of x from the following: | {cc} {x}&{2} {2}&{2x} | = 0 .…
  37. Write the value of the determinant | {ccc} {2}&{3}&{4} {5}&{6}&{8} {6x}&{9x}&{12x} |…
  38. If |A| = 2, where A is 2 × 2 matrix, find |adj A|.
  39. For what is the value of the determinant | {lll} {0}&{2}&{0} {2}&{3}&{4} {4}&{5}&{6}…
  40. For what value of x is the matrix [ {ll} {6-x}&{4} {3-x}&{1} ] singular?…
  41. A matrix A of order 3 × 3 is such that |A| = 4. Find the value of |2A|.…
  42. Evaluate: | {cc} { cos15^{degree } } & { sin15^{circ} } { sin75^{circ} } & {…
  43. If a = [ {lll} {5}&{3}&{8} {2}&{0}&{1} {1}&{2}&{3} ] . Write the cofactor of the…
  44. If | {cc} {x+1}&{x-1} {x-3}&{x+2} | = | {cc} {4}&{-1} {1}&{3} | , then write the…
  45. If | {cc} {2x}&{x+3} { 2 (x+1) } &{x+1} | = | {cc} {1}&{5} {3}&{3} | , then write the…
  46. If | {ll} {3x}&{7} {-2}&{4} | = | {ll} {8}&{7} {6}&{4} | , find the value of x.…
  47. If | {cc} {2x}&{5} {8}&{x} | = | {cc} {6}&{-2} {7}&{3} | , write the value of x.…
  48. If A is a 3 × 3 matrix, |A| ≠ 0 and |3A| = k|A| then write value of k.…
  49. Write the value of the determinant | {cc} {p}&{p+1} {p-1}&{p} | .…
  50. Write the value of the determinant | {ccc} {x+y}&{y+z}&{z+x} {z}&{x}&{y}…
  51. If a = [ {cc} {costheta }&{sintegrate heta} {-sintheta}&{costheta} ] , then for any…
  52. Find the maximum value of | {ccc} {1}&{1}&{1} {1}&{1+sintegrate heta }&{1}…
  53. If x ∈ N and | {cc} {x+3}&{-2} {-3x}&{2x} | = 8 , then find the value of x.…
  54. If | {ccc} {x}&{sintegrate heta }&{costheta } {-sintheta}&{-x}&{1} {costheta}&{1}&{x}…
  55. If A is a 3 × 3 matrix, then what will be the value of k if Det(A-1) = (Det A)k?…

Exercise 6.1
Question 1.

Write the minors and cofactors of each element of the first column of the following matrices and hence evaluate the determinant in each case:



Answer:

Let Mij and Cij represents the minor and co–factor of an element, where i and j represent the row and column.

The minor of the matrix can be obtained for a particular element by removing the row and column where the element is present. Then finding the absolute value of the matrix newly formed.


Also, Cij = (–1)i+j × Mij



M11 = –1


M21 = 20


C11 = (–1)1+1 × M11


= 1 × –1


= –1


C21 = (–1)2+1 × M21


= 20 × –1


= –20


Now expanding along the first column we get


|A| = a11 × C11 + a21× C21


= 5× (–1) + 0 × (–20)


= –5



Question 2.

Write the minors and cofactors of each element of the first column of the following matrices and hence evaluate the determinant in each case:



Answer:

Let Mij and Cij represents the minor and co–factor of an element, where i and j represent the row and column.

The minor of matrix can be obtained for particular element by removing the row and column where the element is present. Then finding the absolute value of the matrix newly formed.


Also, Cij = (–1)i+j × Mij



M11 = 3


M21 = 4


C11 = (–1)1+1 × M11


= 1 × 3


= 3


C21 = (–1)2+1 × 4


= –1 × 4


= –4


Now expanding along the first column we get


|A| = a11 × C11 + a21× C21


= –1× 3 + 2 × (–4)


= –11



Question 3.

Write the minors and cofactors of each element of the first column of the following matrices and hence evaluate the determinant in each case:



Answer:

Let Mij and Cij represents the minor and co–factor of an element, where i and j represent the row and column.

The minor of the matrix can be obtained for a particular element by removing the row and column where the element is present. Then finding the absolute value of the matrix newly formed.


Also, Cij = (–1)i+j × Mij




M11 = –1×2 – 5×2


M11 = –12



M21 = –3×2 – 5×2


M21 = –16



M31 = –3×2 – (–1) × 2


M31 = –4


C11 = (–1)1+1 × M11


= 1 × –12


= –12


C21 = (–1)2+1 × M21


= –1 × –16


= 16


C31 = (–1)3+1 × M31


= 1 × –4


= –4


Now expanding along the first column we get


|A| = a11 × C11 + a21× C21+ a31× C31


= 1× (–12) + 4 × 16 + 3× (–4)


= –12 + 64 –12


= 40



Question 4.

Write the minors and cofactors of each element of the first column of the following matrices and hence evaluate the determinant in each case:



Answer:

Let Mij and Cij represents the minor and co–factor of an element, where i and j represent the row and column.

The minor of the matrix can be obtained for a particular element by removing the row and column where the element is present. Then finding the absolute value of the matrix newly formed.


Also, Cij = (–1)i+j × Mij




M11 = b × ab – c × ca


M11 = ab2 – ac2



M21 = a × ab – c × bc


M21 = a2b – c2b



M31 = a × ca – b × bc


M31 = a2c – b2c


C11 = (–1)1+1 × M11


= 1 × (ab2 – ac2)


= ab2 – ac2


C21 = (–1)2+1 × M21


= –1 × (a2b – c2b)


= c2b – a2b


C31 = (–1)3+1 × M31


= 1 × (a2c – b2c)


= a2c – b2c


Now expanding along the first column we get


|A| = a11 × C11 + a21× C21+ a31× C31


= 1× (ab2 – ac2) + 1 × (c2b – a2b) + 1× (a2c – b2c)


= ab2 – ac2 + c2b – a2b + a2c – b2c



Question 5.

Write the minors and cofactors of each element of the first column of the following matrices and hence evaluate the determinant in each case:



Answer:

Let Mij and Cij represents the minor and co–factor of an element, where i and j represent the row and column.

The minor of matrix can be obtained for particular element by removing the row and column where the element is present. Then finding the absolute value of the matrix newly formed.


Also, Cij = (–1)i+j × Mij




M11 = 5×1 – 7×0


M11 = 5



M21 = 2×1 – 7×6


M21 = –40



M31 = 2×0 – 5×6


M31 = –30


C11 = (–1)1+1 × M11


= 1 × 5


= 5


C21 = (–1)2+1 × M21


= –1 × –40


= 40


C31 = (–1)3+1 × M31


= 1 × –30


= –30


Now expanding along the first column we get


|A| = a11 × C11 + a21× C21+ a31× C31


= 0× 5 + 1 × 40 + 3× (–30)


= 0 + 40 – 90


= 50



Question 6.

Write the minors and cofactors of each element of the first column of the following matrices and hence evaluate the determinant in each case:



Answer:

Let Mij and Cij represents the minor and co–factor of an element, where i and j represent the row and column.

The minor of matrix can be obtained for particular element by removing the row and column where the element is present. Then finding the absolute value of the matrix newly formed.


Also, Cij = (–1)i+j × Mij




M11 = b × c – f × f


M11 = bc– f2



M21 = h × c – f × g


M21 = hc – fg



M31 = h × f – b × g


M31 = hf – bg


C11 = (–1)1+1 × M11


= 1 × (bc– f2)


= bc– f2


C21 = (–1)2+1 × M21


= –1 × (hc – fg)


= fg – hc


C31 = (–1)3+1 × M31


= 1 × (hf – bg)


= hf – bg


Now expanding along the first column we get


|A| = a11 × C11 + a21× C21+ a31× C31


= a× (bc– f2) + h× (fg – hc) + g× (hf – bg)


= abc– af2 + hgf – h2c +ghf – bg2



Question 7.

Write the minors and cofactors of each element of the first column of the following matrices and hence evaluate the determinant in each case:



Answer:

Let Mij and Cij represents the minor and co–factor of an element, where i and j represent the row and column.

The minor of matrix can be obtained for particular element by removing the row and column where the element is present. Then finding the absolute value of the matrix newly formed.


Also, Cij = (–1)i+j × Mij




M11 = 0(–1×0 – 5×1) – 1(1×0 – (–1)×1) + (–2)(1×5 – (–1)×(–1))


M11 = – 9



M21 = –1(–1×0 – 5×1) – 0(1×0 – (–1)×1) + 1(1×5 – (–1)×(–1))


M21 = 9



M31 = –1(1×0 – 5×(–2)) – 0(0×0 – (–1)×(–2)) + 1(0×5 – (–1)×1)


M31 = –9



M41 = –1(1×1 – (–1)×(–2)) – 0(0×1 – 1×(–2)) + 1(0×(–1) – 1×1)


M41 = 0


C11 = (–1)1+1 × M11


= 1 × (–9)


= –9


C21 = (–1)2+1 × M21


= –1 × 9


= –9


C31 = (–1)3+1 × M31


= 1 × –9


= –9


C41 = (–1)4+1 × M41


= –1 × 0


= 0


Now expanding along the first column we get


|A| = a11 × C11 + a21× C21+ a31× C31 + a41× C41


= 2× (–9) + (–3) × –9 + 1× (–9) + 2× 0


= – 18 + 27 –9


= 0



Question 8.

Evaluate the following determinants:

i.

ii.

iii.

iv.


Answer:

I.


⇒ |A| = x(5x + 1) – (–7)x


|A| = 5x2 + 8x


II.


⇒ |A| = cosθ × cosθ – (–sinθ) x sinθ


|A| = cos2θ + sin2θ


|A| = 1


III.


⇒ |A| = cos15° × cos75° + sin15° x sin75°


|A| = cos(75 – 15)°


|A| = cos60°


|A| = 0.5.


IV.


⇒ |A| = (a + ib)( a – ib) – (c + id)( –c + id)


= (a + ib)( a – ib) + (c + id)( c – id)


= a2 – i2 b2 + c2 – i2 d2


= a2 – (–1)b2 + c2 – (–1)d2


= a2 + b2 + c2 + d2



Question 9.

Evaluate



Answer:

Since |AB|= |A||B|



= 2(17×12 – 5×20) – 3(13×12 – 5×15) + 7(13×20 – 15×17)


= 2(204 – 100) – 3(156 – 75) + 7(260 – 255)


= 2×104 – 3×81 + 7×5


= 208 – 243 +35


= 0


Now |A|2 = |A|×|A|


|A|2= 0



Question 10.

Show that


Answer:


Using sin(A+B) = sinA × cosB + cosA × sinB


⇒ |A| = sin10° × cos80° + cos10° x sin80°


|A| = sin(10 + 80)°


|A| = sin90°


|A| = 1


Hence Proved



Question 11.

Evaluate by two methods.


Answer:


I. Expanding along the first row



= 2(1×1 – 4×(–2)) – 3(7×1 – (–2)×(–3)) – 5(7×4 – 1×(–3))


= 2(1 + 8) – 3(7 – 6) – 5(28 + 3)


= 2×9 – 3×1 – 5×31


= 18 – 3 – 155


= –140


II. Expanding along the second column



= 2(1×1 – 4×(–2)) – 7(3×1 – 4×(–5)) – 3(3×(–2) – 1×(–5))


= 2(1 + 8) – 7(3 + 20) – 3(–6 + 5)


= 2×9 – 7×23 – 3×(–1)


= 18 – 161 +3


= –140



Question 12.

Evaluate :


Answer:


Expanding along the first row



⇒ |A| = 0(0 – sinβ(–sinβ) ) –sinα(–sinα× 0 – sinβ cosα ) – cosα((–sinα)(–sinβ) – 0× cosα )


|A| = 0 + sinα sinβ cosα – cosα sinα sinβ


|A| = 0



Question 13.

Evaluate :



Answer:


Expanding along the second row



⇒ |A| = sinβ (cosα×cosα sinβ + sinα × sinα sinβ) + cosβ (cosα cosβ × cosα + sinα×sinα cosβ) – 0


|A| = sin2 β (cos2α + sin2α) + cos2β (cos2α + sin2α)


|A| = sin2 β (1) + cos2β (1)


|A| = sin2 β + cos2β


|A| = 1



Question 14.

If and , verify that |AB| = |A| |B|.


Answer:



Now |A|= 2 × 1 – 2 × 5


|A|= 2 – 10


|A|= –8


Now |B|= 4 × 5 – 2 × (–3)


|B|= 20 + 6


|B|= 26


⇒ |A|×|B|= –8 × 26


|A|×|B|= –208


Now






|AB| = 18 × (–1) – 19 × 10


|AB| = –18 – 190


|AB| = –208


Hence |AB| =|A|×|B|.



Question 15.

If , then show that .


Answer:


Expanding along the first row



= 1(1×4 – 2×0) – 0(0×4 – 0×2) + 1(0×0 – 0×1)


= 1(4 – 0) + 0 + 1(0 + 0)


= 1×4


= 4


Now



Expanding along the first row



= 3(3×12 – 6×0) – 0(0×12 – 0×6) + 3(0×0 – 0×3)


= 3(36 – 0) + 0 + 3(0 + 0)


= 3×36


= 108


= 27 × 4


= 27 |A|


Hence, |3A|= 27 |A|



Question 16.

Find the value of x, if



Answer:


⇒ 2 × 1 – 4 × 5 = 2x × x – 4 × 6


⇒ 2 – 20 = 2x2 – 24


⇒ 2x2 = –18 +24


⇒ 2x2 = 6


⇒ x2 = 3


⇒ x = ±√ 3



Question 17.

Find the value of x, if



Answer:


⇒ 2 × 5 – 4 × 3 = x × 5 – 2x × 3


⇒ 10 – 12 = 5x – 6x


⇒ –x = –2


⇒ x = 2



Question 18.

Find the value of x, if



Answer:


⇒ 3 × 1 – x × x = 3 × 1 – 4 × 2


⇒ 3 – x2 = 3 – 8


⇒ – x2 = –5 –3


⇒ –x2 = –8


⇒ x = ±2√ 2



Question 19.

Find the value of x, if



Answer:


⇒ 3x × 4 – 7 × 2 = 10


⇒ 12x – 14 = 10


⇒ 12x= 10 +14


⇒ 12x = 24


⇒ x = 2



Question 20.

Find the value of x, if



Answer:


⇒ (x+1)(x+2) – (x–1)(x–3) = 4 × 3 – 1 × (–1)


⇒ (x2 + 2x + x + 2) – (x2 – 3x – x + 3) = 12 + 1


⇒ –2x –1= 13


⇒ –2x = 14


⇒ x = –7



Question 21.

Find the value of x, if



Answer:


⇒ 2x × x – 5 × 8 = 6 × 3 – 5 × 8


⇒ 2x2 – 40 = 18 – 40


⇒ 2x2 = 18


⇒ x2 = 9


⇒ x = ±3



Question 22.

Find the integral value of x, if


Answer:


Expanding along the first row



= x2(2×4 – 1×1) – x(0×4 – 1×3) + 1(0×1 – 2×3)


= x2(8 – 1) – x(0 – 3) + 1(0 – 6)


= 7x2 + 3x –6


Also |A| = 28


⇒ 7x2 + 3x – 6 =28


⇒ 7x2 + 3x – 34 = 0


⇒ 7x2 + 17x – 14x – 34 = 0


⇒ x(7x+ 17) – 2(7x +17) = 0


⇒ (x–2)(7x +17) = 0



Integer value of x is 2.



Question 23.

For what value of x matrix A is singular?



Answer:



⇒ (1 + x) × 8 – 7 × (3 – x) = 0


⇒ 8 + 8x – 21 + 7x = 0


⇒ 15x – 13 = 0




Question 24.

For what value of x matrix A is singular?



Answer:


Expanding along the first row



= (x–1) ((x–1) (x–1)– 1×1) – 1((x–1) – 1×1) + 1(1×1 – 1×(x–1))


= (x–1) (x2 – 2x + 1 – 1) – 1(x–1 – 1) + 1(1 – x+1)


= x(x–1) (x– 2) – 1(x–2) – (x–2)


= (x– 2) {x(x–1) – 1 – 1}


= (x– 2) (x2 – x – 2)


For singular |A| = 0,


(x– 2) (x2 – x – 2) = 0


(x– 2) (x2 – 2x + x – 2) = 0


(x–2)(x–2)(x+1) = 0


∴ x = –1 or 2


Also |A| = 28


⇒ 7x2 + 3x – 6 =28


⇒ 7x2 + 3x – 34 = 0


⇒ 7x2 + 17x – 14x – 34 = 0


⇒ x(7x+ 17) – 2(7x +17) = 0


⇒ (x–2)(7x +17) = 0




Exercise 6.2
Question 1.

Evaluate the following determinant:



Answer:


Applying, R2→ R2 – R1, we get,



So,



Question 2.

Evaluate the following determinant:



Answer:


Applying, C1 → C1 – 4 C3, we get,



Applying, R1 → R1 + R2 and R3→ R3 – R2, we get



Now, applying R2 → R2 + 3 R1, we get,



= 1[(109)(12) – (119)(11)] = 1308 – 1309


= – 1


So, Δ = – 1



Question 3.

Evaluate the following determinant:



Answer:


= a(bc – f2) – h(hc – fg) + g(hf – bg)


= abc – af2 – ch2 + fgh + fgh – bg2


= abc + 2fgh – af2 – bg2 – ch2


So, Δ = abc + 2fgh – af2 – bg2 – ch2



Question 4.

Evaluate the following determinant:



Answer:



Applying, R2 → R2 – R1 and R3 → R3 – R1, we get



= 2[1(24 – 4)] = 40


So, Δ = 40



Question 5.

Evaluate the following determinant:



Answer:


Applying C3→ C3 – C2, we get,



Applying C2 → C2 + C1, we get,



Applying C2→ C2 – 5C1 and C3 →C3 – 5C1 we get,



= 1[( – 7)( – 36) – ( – 20)( – 13)] = 252 – 260


= – 8


So, Δ = – 8



Question 6.

Evaluate the following determinant:



Answer:


Applying, R1 → R1 – 3R2 and R3 → R3 + 5R2 we get,



So, Δ = 0



Question 7.

Evaluate the following determinant:



Answer:



Applying C1 → C1 + C2 + C3 + C4, we get,




Now, applying R2 → R2 – R1, R3 → R3 – R1, R4 → R4 – R1, we get





Now, applying R1 → R1 + 3R3




= (40)(8)(40)(40) = 512000


So, Δ = 512000



Question 8.

Evaluate the following determinant:



Answer:



Applying R3 → R3 – R1, we get,



So, Δ = 0



Question 9.

Without expanding, show that the value of each of the following determinants is zero:



Answer:


Applying R3 → R3 – R2, we get



Applying R2 → R2 – R1, we get



As, R2 = R3, therefore the value of the determinant is zero.



Question 10.

Without expanding, show that the value of each of the following determinants is zero:



Answer:


Taking ( – 2) common from C1 we get,



As, C1 = C2, hence the value of the determinant is zero.



Question 11.

Without expanding, show that the value of each of the following determinants is zero:



Answer:


Applying C3 → C3 – C2, gives



As, R1 = R3, so value so determinant is zero.



Question 12.

Without expanding, show that the value of each of the following determinants is zero:



Answer:


Multiplying R1, R2 and R3 with a, b and c respectively we get,



Taking, abc common from C3 gives,



As, C1 = C3 hence value of determinant is zero.



Question 13.

Without expanding, show that the value of each of the following determinants is zero:



Answer:


Applying C3 → C3 – C2, we get,



Applying C2→C2 – C1 gives,



As, C2 = C3, so the value of the determinant is zero.



Question 14.

Without expanding, show that the value of each of the following determinants is zero:



Answer:



Applying R2→R2 – R1 and R3 → R3 – R1, we get,



Taking (b – a) and (c – a) common from R2 and R3 respectively,



= [(b – a)(c – a)][(c + a) – (b + a) – ( – b + c)]


= [(b – a)(c – a)][c + a + b – a – b – c]


= [(b – a)(c – a)][0] = 0



Question 15.

Without expanding, show that the value of each of the following determinants is zero:



Answer:


Applying, C1→C1 – 8C3



As, C1 = C2 hence, the determinant is zero.



Question 16.

Without expanding, show that the value of each of the following determinants is zero:



Answer:


Multiplying C1, C2 and C3 with z, y and x respectively we get,



Now, taking y, x and z common from R1, R2 and R3 gives,



Applying C2 → C2 – C3 gives,



As, C1 = C2, therefore determinant is zero.



Question 17.

Without expanding, show that the value of each of the following determinants is zero:



Answer:


Applying C2→C2 – 7C3, we get



As, C1 = C2, hence determinant is zero.



Question 18.

Without expanding, show that the value of each of the following determinants is zero:



Answer:


Applying C3→C3 – C2, and C4→C4 – C1




Taking 3 common from C4 we get,



As, C3 = C4 so, the determinant is zero.



Question 19.

Without expanding, show that the value of each of the following determinants is zero:



Answer:


Applying, C2→C2 + C1 and C3→C3 + C1



Taking 2 common from R2 we get,



As, R2 = R3, hence value of determinant is zero.



Question 20.

Without expanding, show that the value of each of the following determinants is zero:



Answer:



Applying, C1→C1 – C2, we get




As C1 = C3 hence determinant is zero.



Question 21.

Without expanding, show that the value of each of the following determinants is zero:



Answer:


Multiplying C1 with sin δ, C2 with cos δ, we get



Now, applying, C2→C2 – C1, we get,




As C2 = C3 hence determinant is zero.



Question 22.

Without expanding, show that the value of each of the following determinants is zero:



Answer:


Applying C1→C1 + C2, we get



Using, sin(90 – A) = cos A, sin2 A + cos2 A = 1 ,and cos 180˚ = – 1,




Taking, ( – 1) common from C1, we get



Therefore, as C1 = C3 determinant is zero.



Question 23.

Without expanding, show that the value of each of the following determinants is zero:



Answer:


Multiplying R2 with sin y and R3 with cos y we get,



Now, applying R2→R2 + R3, we get,



Taking ( – 1) common from R2, we get




As R1 = R2 hence determinant is zero.



Question 24.

Without expanding, show that the value of each of the following determinants is zero:



Answer:


Multiplying C2 with and C3 with we get,




Taking common from C2 and C3 we get,



Applying C2→C2 + C3



As C1 = C2 hence determinant is zero.



Question 25.

Without expanding, show that the value of each of the following determinants is zero:

, where A, B, C are the angles of ΔABC.


Answer:


Now,


Δ = sin2 A (cot B – cot C) – cot A (sin2 B – sin2 C) + 1 (sin2B cot C – cot B sin2 C


As A, B and C are angles of a triangle,


A + B + C = 180°


Δ = sin2 A cot B – sin2 A cot C – cot A sin2 B + cot A sin2C + sin2B cot C – cot B sin2 C


By using formulae,




Δ = 0


Hence, Proved.



Question 26.

Evaluate the following:



Answer:


Applying, C2→C2 + C1



Taking, (a + b + c) common,



Applying R2→R2 – R1, and R3→R3 – R1



Taking, (b – c) and (c – a) common,



= (a + b + c)(b – a)(c – a)(b – c)


So, Δ = (a + b + c)(b – a)(c – a)(b – c)



Question 27.

Evaluate the following:



Answer:


Applying, R2→R2 – R1 and R3→R3 – R1 we get,




Taking (a – b) and (a – c) common we get,



= (a – b)(c – a)(b – c)


So, Δ = (a – b)(b – c)(c – a)



Question 28.

Evaluate the following:



Answer:


Applying, C1→C1 + C2 + C3, we have,



Taking, (3x + λ) common, we get



Applying, R2→R2 – R1, R3→R3 – R1, we get,



= λ2(3x + λ)


So, Δ = λ2(3x + λ)



Question 29.

Evaluate the following:



Answer:


Applying, C1→C1 + C2 + C3, we get,



Taking, (a + b + c) we get,



Applying, R2→R2 – R1, R3→R3 – R1, we get,



= (a + b + c)[(a – b)(a – c) – (b – c)(c – b)]


= (a + b + c)[a2 – ac – ab + bc + b2 + c2 – 2bc]


= (a + b + c)[a2 + b2 + c2 – ac – ab – bc]


So, Δ = (a + b + c)[a2 + b2 + c2 – ac – ab – bc]



Question 30.

Evaluate the following:



Answer:


Applying, C1→C1 + C2 + C3, we get,




Applying, R2→R2 – R1, R3→R3 – R1, we get,



= (2 + x)(x – 1)2


So, Δ = (2 + x)(x – 1)2



Question 31.

Evaluate the following:



Answer:


= 0(0 – y3z3) – xy2(0 – x2yz3) + xz2(x2y3z – 0)


= 0 + x3y3z3 + x3y3z3


= 2x3y3z3


So, Δ = 2x3y3z3



Question 32.

Evaluate the following:



Answer:


Applying R1→R1 – R2 and R3→R3 – R2



Applying, C2→C2 – C1



= a[a(a + x + y) + az] + 0 + 0


= a2(a + x + y + z)


So, Δ = a2(a + x + y + z)



Question 33.

If ,, then prove that Δ + Δ1 = 0.


Answer:


As |A| = |A|T



If any two rows or columns of the determinant are interchanged, then determinant changes its sign




So, Δ = 0



Question 34.

Prove the following identities:



Answer:


L.H.S =


Apply C1→C1 + C2 + C3



Taking (a + b + c) common from C1 we get,



Applying, R3→R3 – 2R1



= (a + b + c)[(b – c)(a + b – 2c) – (c – a)(c + a – 2b)]


= a3 + b3 + c3 – 3abc


As, L.H.S = R.H.S


Hence, proved.



Question 35.

Prove the following identities:



Answer:

L.H.S =


As |A| = |A|T


So,


If any two rows or columns of the determinant are interchanged, then determinant changes its sign



Apply C1→C1 + C2 + C3



Taking (a + b + c) common from C1 we get,



Applying, R3→R3 – 2R1



= – (a + b + c)[(b – c)(a + b – 2c) – (c – a)(c + a – 2b)]


= 3abc – a3 – b3 – c3


As, L.H.S = R.H.S, hence proved.



Question 36.

Prove the following identities:



Answer:


L.H.S =


Applying, C1→C1 + C2 + C3




Apply, C2→C2 – C1, and C3→C3 – C1, we have







Hence, proved.



Question 37.

Prove the following identities:



Answer:

,


R.H.S = 2(a + b + c)2


Applying C1→C1 + C2 + C3, we have



Taking, 2(a + b + c) common we get,



Now, applying R2→R2 – R1 and R3→R3 – R1, we get,



Thus, we have


L.H.S = 2(a + b + c)[1(a + b + c)2]


= 2(a + b + c)3 = R.H.S


Hence, proved.



Question 38.

Prove the following identities:



Answer:

L.H.S =


Applying, R1→R1 + R2 + R3, we get,



Taking (a + b + c) common we get,



Applying C2→C2 – C1 and C3→C3 – C1, we get,




= (a + b + c)3 = R.H.S


Hence, proved.



Question 39.

Prove the following identities:



Answer:


Applying, R2→R2 – R1 and R3→R3 – R1, we get,




Applying R3→R3 – R2, we get,



= (a – b)(a – c)(b – c) = R.H.S


Hence, proved.



Question 40.

Prove the following identities:



Answer:


Applying R1→R1 + R2 + R3, we get,



Taking, (3a + 2b) common we get,



Applying, C1→C1 – C2 and C3→C3 – C2, we get,




= 3(a + b)b2(3) = 9(a + b) b2


= R.H.S


Hence, proved.



Question 41.

Prove the following identities:



Answer:


Applying, R1→a R1, R2→b R2, R3→c R3






Hence, proved.



Question 42.

Prove the following identities:

=


Answer:






= – xyz(x – y)(z – y)[z2 + y2 + zy – x2 – y2 – xy]


= – xyz(x – y)(z – y)[(z – x)(z + x0 + y(z – x)]


= – xyz(x – y)(z – y)(z – x)(x + y + z)


= R.H.S


Hence, proved.



Question 43.

Prove the following identities:





Answer:


Applying, C1→C1 + C2 – 2C3




Taking (a2 + b2 + c2), common, we get,



Applying R2→R2 – R1 and R3→R3 – R1, we get,




= (a2 + b2 + c2)(b – a)(c – a)[(b + a)( – b) – ( – c)(c + a)]


= (a2 + b2 + c2)(a – b)(c – a)(b – c)(a + b + c)


= R.H.S


Hence, proved.



Question 44.

Prove the following identities:



Answer:


Applying, R3→R3 – R2



Applying, R2→R2 – R1



= [(2a + 4)(1) – (1)(2a + 6)]


= – 2


= R.H.S


Hence, proved.



Question 45.

Prove the following identities:



Answer:


Applying, C2→C2 – 2C1 – 2C3, we get,




Taking, – (a2 + b2 + c2) common from C2 we get,



Applying R2→R2 – R1 and R3→R3 – R1, we get




= – (a2 + b2 + c2)(a – b)(c – a)[( – (b + a))( – b) – (c)(c + a)]


= (a – b)(b – c)(c – a)(a + b + c)(a2 + b2 + c2)


= R.H.S


Hence, proved.



Question 46.

Prove the following identities:





Answer:


Applying, R2→R2 – R1, and R3→R3 – R1





= (b – a)(c – a)[((b + a – c))(c2 + a2 + ac) – (b2 + a2 + ab)(c2 + a2 + ac)]


= – (a – b)(c – a)(b – c)(a2 + b2 + c2)


= R.H.S


Hence, proved.



Question 47.

Prove the following identities:



Answer:


Taking, a,b,c common from C1, C2, C3 respectively we get,



Applying, C1→C1 + C2 + C3, we get,




Applying, C2→C2 – C1 and C3→C3 – C1, we get,



Applying, C1→C1 + C2 + C3, we get,



Taking c, a, b common from C1, C2, C3 respectively, we get,



Applying, R3→R3 – R1, we have



= 2a2b2c2(2)


= 4a2b2c2 = R.H.S


Hence, proved.



Question 48.

Prove the following identities:



Answer:


Applying, C1→C1 + C2 + C3, we get,



Taking (3x + 4) common we get,



Applying, R2→R2 – R1 and R3→R3 – R1, we get,



= 16(3x + 4)


Hence proved.



Question 49.

Prove the following identities –



Answer:

Let


Recall that the value of a determinant remains same if we apply the operation Ri→ Ri + kRj or Ci→ Ci + kCj.


Applying C2→ C2 – pC1, we get




Applying C3→ C3 – qC1, we get




Applying C3→ C3 – pC2, we get




Applying C2→ C2 – C1, we get




Applying C3→ C3 – C1, we get




Expanding the determinant along R1, we have


Δ = 1[(1)(7) – (3)(2)] – 0 + 0


∴ Δ = 7 – 6 = 1


Thus,



Question 50.

Prove the following identities –

= (a + b - c)(b + c - a)(c + a - b)


Answer:

Let


Recall that the value of a determinant remains same if we apply the operation Ri→ Ri + kRj or Ci→ Ci + kCj.


Applying R1→ R1 – R2, we get




Applying R1→ R1 – R3, we get





Taking the term (a – b – c) common from R1, we get



Applying C2→ C2 + C1, we get




Applying C3→ C3 + C1, we get




Expanding the determinant along R1, we have


Δ = (a – b – c)[–1(b + a – c)(c + a – b) – 0 + 0]


⇒ Δ = –(a – b – c)(b + a – c)(c + a – b)


∴ Δ = (b + c – a)(a + b – c)(c + a – b)


Thus,



Question 51.

Prove the following identities –



Answer:

Let


Recall that the value of a determinant remains same if we apply the operation Ri→ Ri + kRj or Ci→ Ci + kCj.


Applying R1→ R1 + R2, we get



Applying R1→ R1 + R3, we get




Taking the term (a2 + b2 + 2ab) common from R1, we get



Applying C2→ C2 – C1, we get




Applying C3→ C3 – C1, we get




Expanding the determinant along R1, we have


Δ = (a + b)2 [(a2 – b2)(a2 – 2ab) – (b2 – 2ab)(2ab – b2)]


⇒ Δ = (a + b)2 [a4 – 2a3b – b2a2 + 2ab3 – 2ab3 + b4 + 4a2b2 – 2ab3]


⇒ Δ = (a + b)2 [a4 – 2a3b + 3a2b2 – 2ab3 + b4]


⇒ Δ = (a + b)2 [a4 + b4 + 2a2b2 – 2a3b – 2ab3 + a2b2]


⇒ Δ = (a + b)2 [(a2 + b2)2 – 2ab(a2 + b2) + (ab)2]


⇒ Δ = (a + b)2 [(a2 + b2 – ab)2]


⇒ Δ = [(a + b)(a2 + b2 – ab)]2


∴ Δ = (a3 + b3)2


Thus,



Question 52.

Prove the following identities –



Answer:

Let


Recall that the value of a determinant remains same if we apply the operation Ri→ Ri + kRj or Ci→ Ci + kCj.


Applying R1→ R1 + R2, we get



Applying R1→ R1 + R3, we get




Taking the term (a2 + a + 1) common from R1, we get



Applying C2→ C2 – C1, we get




Applying C3→ C3 – C1, we get




Expanding the determinant along R1, we have


Δ = (a2 + a + 1)(1)[(1 – a2)(1 – a) – (a2 – a)(a – a2)]


⇒ Δ = (a2 + a + 1)(1 – a – a2 + a3 – a3 + a4 + a2 – a3)


⇒ Δ = (a2 + a + 1)(1 – a – a3 + a4)


⇒ Δ = (a2 + a + 1)(a4 – a3 – a + 1)


⇒ Δ = (a2 + a + 1)[a3(a – 1) – (a – 1)]


⇒ Δ = (a2 + a + 1)(a – 1)(a3 – 1)


⇒ Δ = (a – 1)(a2 + a + 1)(a3 – 1)


⇒ Δ = (a3 – 1)(a3 – 1)


∴ Δ = (a3 – 1)2


Thus,



Question 53.

Prove the following identities –

= 2(a + b)(b + c)(c + a)


Answer:

Let


Recall that the value of a determinant remains same if we apply the operation Ri→ Ri + kRj or Ci→ Ci + kCj.


Applying R1→ R1 + R2, we get




Applying R1→ R1 + R3, we get




Applying C2→ C2 + C1, we get




Applying C3→ C3 + C1, we get




Taking (a + b) and (c + a) common from C2 and C3, we get



Applying R2→ R2 + R1, we get




Applying R3→ R3 – R1, we get




Expanding the determinant along C3, we have


Δ = (a + b)(c + a)[(–c + a)(–2) – (–b – a)(2)]


⇒ Δ = (a + b)(c + a)[2c – 2a + 2a + 2b]


⇒ Δ = (a + b)(c + a)(2b + 2c)


∴ Δ = 2(a + b)(b + c)(c + a)


Thus,



Question 54.

Prove the following identities –



Answer:

Let


Recall that the value of a determinant remains same if we apply the operation Ri→ Ri + kRj or Ci→ Ci + kCj.


Applying R1→ R1 – R2, we get




Applying R1→ R1 – R3, we get




Applying C2→ C2 – C1, we get




Applying C3→ C3 – C1, we get




Expanding the determinant along R1, we have


⇒ Δ = 0 + (2c)[(b)(a + b – c)] + (–2b)[–(c)(c + a – b)]


⇒ Δ = 2bc(a + b – c) + 2bc(c + a – b)


⇒ Δ = 2bc[(a + b – c) + (c + a – b)]


⇒ Δ = 2bc[2a]


∴ Δ = 4abc


Thus,



Question 55.

Prove the following identities –



Answer:

Let


Multiplying a, b and c to R1, R2 and R3, we get




Dividing C1, C2 and C3 with a, b and c, we get



Recall that the value of a determinant remains same if we apply the operation Ri→ Ri + kRj or Ci→ Ci + kCj.


Applying R1→ R1 – R2, we get




Applying R1→ R1 – R3, we get




Applying C2→ C2 – C1, we get




Applying C3→ C3 – C1, we get




Expanding the determinant along R1, we have


Δ = 0 + (2c2)[(b2)(a2 + b2 – c2)] + (–2b2)[–(c2)(c2 + a2 – b2)]


⇒ Δ = 2b2c2(a2 + b2 – c2) + 2b2c2(c2 + a2 – b2)


⇒ Δ = 2b2c2 [(a2 + b2 – c2) + (c2 + a2 – b2)]


⇒ Δ = 2b2c2[2a2]


∴ Δ = 4a2b2c2


Thus,



Question 56.

Prove the following identities –



Answer:

Let


Taking a2, b2 and c2 common from C1, C2 and C3, we get



Taking a, b and c common from R1, R2 and R3, we get



Recall that the value of a determinant remains same if we apply the operation Ri→ Ri + kRj or Ci→ Ci + kCj.


Applying C2→ C2 – C3, we get




Expanding the determinant along R1, we have


Δ = (a3b3c3)[0 – 0 + 1(1)(1) – (1)(–1)]


⇒ Δ = (a3b3c3)[1 + 1]


∴ Δ = 2a3b3c3


Thus,



Question 57.

Prove the following identities –



Answer:

Let


Multiplying c, a and b to R1, R2 and R3, we get



Recall that the value of a determinant remains same if we apply the operation Ri→ Ri + kRj or Ci→ Ci + kCj.


Applying R1→ R1 – R2, we get




Applying R1→ R1 – R3, we get




Applying C2→ C2 – C1, we get




Applying C3→ C3 – C1, we get




Expanding the determinant along R1, we have







∴ Δ = 4abc


Thus,



Question 58.

Prove the following identities –



Answer:

Let


Multiplying a, b and c to R1, R2 and R3, we get




Dividing C1, C2 and C3 with a, b and c, we get



Recall that the value of a determinant remains same if we apply the operation Ri→ Ri + kRj or Ci→ Ci + kCj.


Applying R1→ R1 + R2, we get




Applying R1→ R1 + R3, we get




Taking the term (a – b – c) common from R1, we get



Applying C2→ C2 – C1, we get




Applying C3→ C3 – C1, we get




Expanding the determinant along R1, we have


Δ = (ab + bc + ca)(1)[(ab + bc + ca)(ab + bc + ca)]


∴ Δ = (ab + bc + ca)3


Thus,



Question 59.

Prove the following identities –



Answer:

Let


Recall that the value of a determinant remains same if we apply the operation Ri→ Ri + kRj or Ci→ Ci + kCj.


Applying R1→ R1 + R2, we get




Applying R1→ R1 + R3, we get




Taking the term (5x + λ) common from R1, we get



Applying C2→ C2 – C1, we get




Applying C3→ C3 – C1, we get




Expanding the determinant along R1, we have


Δ = (5x + λ)[(1)(λ – x)(λ – x)]


∴ Δ = (5x + λ)(λ – x)2


Thus,



Question 60.

Prove the following identities –



Answer:

Let


Recall that the value of a determinant remains same if we apply the operation Ri→ Ri + kRj or Ci→ Ci + kCj.


Applying R1→ R1 + R2, we get




Applying R1→ R1 + R3, we get




Taking the term (5x + 4) common from R1, we get



Applying C2→ C2 – C1, we get




Applying C3→ C3 – C1, we get




Expanding the determinant along R1, we have


Δ = (5x + 4)[(1)(4 – x)(4 – x)]


∴ Δ = (5x + 4)(4 – x)2


Thus,



Question 61.

Prove the following identities –



Answer:

Let


Recall that the value of a determinant remains same if we apply the operation Ri→ Ri + kRj or Ci→ Ci + kCj.


Applying R1→ R1 – R2, we get




Applying R1→ R1 – R3, we get




Taking the term (–2x) common from R1, we get



Applying C2→ C2 – C3, we get




Expanding the determinant along R1, we have


Δ = (–2x)[(z)(–y) – (y)(z)]


⇒ Δ = (–2x)(–yz –yz)


⇒ Δ = (–2x)(–2yz)


∴ Δ = 4xyz


Thus,



Question 62.

Prove the following identities –

= abc(a2 + b2 + c2)3


Answer:

Let


Taking a, b and c common from C1, C2 and C3, we get


Recall that the value of a determinant remains same if we apply the operation Ri→ Ri + kRj or Ci→ Ci + kCj.


Applying R1→ R1 – R2, we get




Taking the term (a2 + b2 + c2) common from R1, we get



Applying R2→ R2 – R3, we get




Taking the term (a2 + b2 + c2) common from R2, we get



Applying C2→ C2 + C1, we get




Expanding the determinant along R1, we have


Δ = (abc)(a2 + b2 + c2)2(–1)[(a2 + b2 – c2) – (2b2 + 2a2)]


⇒ Δ = (abc)(a2 + b2 + c2)2[–(a2 + b2 – c2) + (2b2 + 2a2)]


⇒ Δ = (abc)(a2 + b2 + c2)2[–a2 – b2 + c2 + 2b2 + 2a2]


⇒ Δ = (abc)(a2 + b2 + c2)2[a2 + b2 + c2]


∴ Δ = (abc)(a2 + b2 + c2)3


Thus,



Question 63.

Prove the following identities –



Answer:

Let


Recall that the value of a determinant remains same if we apply the operation Ri→ Ri + kRj or Ci→ Ci + kCj.


Applying R1→ R1 + R2, we get




Applying R1→ R1 + R3, we get




Taking the term (3 + a) common from R1, we get



Applying C2→ C2 – C1, we get




Applying C3→ C3 – C1, we get




Expanding the determinant along R1, we have


Δ = (3 + a)(1)[(a)(a) – 0]


⇒ Δ = (3 + a)(a2)


∴ Δ = a3 + 3a2


Thus,



Question 64.

Prove the following identities –



Answer:

Let


Recall that the value of a determinant remains same if we apply the operation Ri→ Ri + kRj or Ci→ Ci + kCj.


Applying R1→ R1 + R2, we get




Applying R1→ R1 + R3, we get




Taking the term (x + y + z) common from R1, we get



Applying C2→ C2 – C1, we get




Applying C3→ C3 – C1, we get




Expanding the determinant along R1, we have


Δ = (x + y + z)(1)[0 – (–(x + y + z)(x + y + z))]


⇒ Δ = (x + y + z)(x + y + z)(x + y + z)


∴ Δ = (x + y + z)3


Thus,



Question 65.

Prove the following identities –



Answer:

Let


Recall that the value of a determinant remains same if we apply the operation Ri→ Ri + kRj or Ci→ Ci + kCj.


Applying R1→ R1 + R2, we get




Applying R1→ R1 + R3, we get




Taking the term (x + y + z) common from R1, we get



Applying C1→ C1 – C2, we get




Applying C1→ C1 – C3, we get




Expanding the determinant along C1, we have


Δ = (x + y + z)(x – z)[(1)(x) – (z)(1)]


⇒ Δ = (x + y + z)(x – z)(x – z)


∴ Δ = (x + y + z)(x – z)2


Thus,



Question 66.

Prove the following identities –



Answer:

Let


Recall that the value of a determinant remains same if we apply the operation Ri→ Ri + kRj or Ci→ Ci + kCj.


Applying C1→ C1 + C2, we get




Applying C1→ C1 + C3, we get




Taking the term (a + x + y + z) common from C1, we get



Applying R2→ R2 – R1, we get




Applying R3→ R3 – R1, we get




Expanding the determinant along C1, we have


Δ = (a + x + y + z)(1)[(a)(a) – (0)(0)]


⇒ Δ = (a + x + y + z)(a)(a)


∴ Δ = a2(a + x + y + z)


Thus,



Question 67.

Prove the following identities –

= 2(a - b)(b - c)(c - a)(a + b + c)


Answer:

Let


Taking 2 common from C2, we get



Recall that the value of a determinant remains same if we apply the operation Ri→ Ri + kRj or Ci→ Ci + kCj.


Applying R2→ R2 – R1, we get




Applying R3→ R3 – R1, we get




We have the identity a3 – b3 = (a – b)(a2 + ab + b2)



Taking (b – a) and (c – a) common from R2 and R3, we get



We know that the sign of a determinant changes if any two rows or columns are interchanged.


By interchanging C1 and C2, we get



Expanding the determinant along C1, we have


Δ = –2(b – a)(c – a)(1)[(b2 + ba + a2) – (c2 + ca + a2)]


⇒ Δ = 2(a – b)(c – a)[b2 + ba + a2 – c2 – ca – a2]


⇒ Δ = 2(a – b)(c – a)[b2 + ba – c2 – ca]


⇒ Δ = 2(a – b)(c – a)[b2 – c2 + (ba – ca)]


⇒ Δ = 2(a – b)(c – a)[(b – c)(b + c) + (b – c)a]


⇒ Δ = 2(a – b)(c – a)(b – c)(b + c + a)


∴ Δ = 2(a – b)(b – c)(c – a)(a + b + c)


Thus,



Question 68.

Without expanding, prove that .


Answer:

Let


We know that the sign of a determinant changes if any two rows or columns are interchanged.


By interchanging R1 and R2, we get



By interchanging R2 and R3, we get




Hence,


Let us once again consider


By interchanging R1 and R2, we get



By interchanging C1 and C2, we get




Recall that the value of a determinant remains same if it its rows and columns are interchanged.



Hence,


Thus,



Question 69.

Show that where a,b,c are in A.P.


Answer:

Let


Recall that the value of a determinant remains same if we apply the operation Ri→ Ri + kRj or Ci→ Ci + kCj.


Applying R1→ R1 + R3, we get




Given that a, b and c are in an A.P. Using the definition of an arithmetic progression, we have


b – a = c – b


⇒ b + b = c + a


⇒ 2b = c + a


∴ a + c = 2b


By substituting this in the above equation to find Δ, we get




Taking 2 common from R1, we get



Applying R1→ R1 – R2, we get




∴ Δ = 0


Thus, when a, b and c are in A.P.



Question 70.

Show that where α, β and γ are in A.P.


Answer:

Let


Recall that the value of a determinant remains same if we apply the operation Ri→ Ri + kRj or Ci→ Ci + kCj.


Applying R1→ R1 + R3, we get




Given that α, β and γ are in an A.P. Using the definition of an arithmetic progression, we have


β – α = γ – β


⇒ β + β = γ + α


⇒ 2β = γ + α


∴ α + γ = 2β


By substituting this in the above equation to find Δ, we get




Taking 2 common from R1, we get



Applying R1→ R1 – R2, we get




∴ Δ = 0


Thus, when α, β and γ are in A.P.



Question 71.

If a, b, c are real numbers such that , then show that either a + b + c = 0 or a = b = c.


Answer:

Let


Given that Δ = 0.


Recall that the value of a determinant remains same if we apply the operation Ri→ Ri + kRj or Ci→ Ci + kCj.


Applying R1→ R1 + R2, we get




Applying R1→ R1 + R3, we get




Taking the term 2(a + b + c) common from R1, we get



Applying C2→ C2 – C1, we get




Applying C3→ C3 – C1, we get




Expanding the determinant along R1, we have


Δ = 2(a + b + c)(1)[(b – c)(c – b) – (c – a)(b – a)]


⇒ Δ = 2(a + b + c)(bc – b2 – c2 + cb – cb + ca + ab – a2)


∴ Δ = 2(a + b + c)(ab + bc + ca – a2 – b2 – c2)


We have Δ = 0


⇒ 2(a + b + c)(ab + bc + ca – a2 – b2 – c2) = 0


⇒ (a + b + c)(ab + bc + ca – a2 – b2 – c2) = 0


Case – I:


a + b + c = 0


Case – II:


ab + bc + ca – a2 – b2 – c2 = 0


⇒ a2 + b2 + c2 – ab – bc – ca = 0


Multiplying 2 on both sides, we have


2(a2 + b2 + c2 – ab – bc – ca) = 0


⇒ 2a2 + 2b2 + 2c2 – 2ab – 2bc – 2ca = 0


⇒ a2 – 2ab + b2 + b2 – 2bc + c2 + c2 – 2ca + a2 = 0


⇒ (a – b)2 + (b – c)2 + (c – a)2 = 0


We know (a – b)2 ≥ 0, (b – c)2 ≥ 0, (c – a)2 ≥ 0


If the sum of three non-negative numbers is zero, then each of the numbers is zero.


⇒ (a – b)2 = 0 = (b – c)2 = (c – a)2


⇒ a – b = 0 = b – c = c – a


⇒ a = b = c


Thus, if, then either a + b + c = 0 or a = b = c.



Question 72.

If , find the value of .


Answer:

Let


Given that Δ = 0.


Recall that the value of a determinant remains same if we apply the operation Ri→ Ri + kRj or Ci→ Ci + kCj.


Applying R1→ R1 – R2, we get




Applying R2→ R2 – R3, we get




Expanding the determinant along R1, we have


Δ = (p – a)[(q – b)(r) – (b)(c – r)] – (b – q)[–a(c – r)]


⇒ Δ = r(p – a)(q – b) – b(p – a)(c – r) + a(b – q)(c – r)


∴ Δ = r(p – a)(q – b) + b(p – a)(r – c) + a(q – b)(r – c)


We have Δ = 0


⇒ r(p – a)(q – b) + b(p – a)(r – c) + a(q – b)(r – c) = 0


On dividing the equation with (p – a)(q – b)(r – c), we get










Thus,



Question 73.

Show that x = 2 is a root of the equation and solve it completely.


Answer:

Let


We need to find the roots of Δ = 0.


Recall that the value of a determinant remains same if we apply the operation Ri→ Ri + kRj or Ci→ Ci + kCj.


Applying R2→ R2 – R1, we get





Taking the term (x – 2) common from R2, we get



Applying R3→ R3 – R1, we get





Taking the term (x + 3) common from R3, we get



Applying C1→ C1 + C3, we get




Expanding the determinant along C1, we have


Δ = (x – 2)(x + 3)(x – 1)[(–3)(1) – (2)(1)]


⇒ Δ = (x – 2)(x + 3)(x – 1)(–5)


∴ Δ = –5(x – 2)(x + 3)(x – 1)


The given equation is Δ = 0.


⇒ –5(x – 2)(x + 3)(x – 1) = 0


⇒ (x – 2)(x + 3)(x – 1) = 0


Case – I:


x – 2 = 0 ⇒ x = 2


Case – II:


x + 2 = 0 ⇒ x = –3


Case – III:


x – 1 = 0 ⇒ x = 1


Thus, 2 is a root of the equation and its other roots are –3 and 1.



Question 74.

Solve the following determinant equations:



Answer:


Let


We need to find the roots of Δ = 0.


Recall that the value of a determinant remains same if we apply the operation Ri→ Ri + kRj or Ci→ Ci + kCj.


Applying C1→ C1 + C2, we get




Applying C1→ C1 + C3, we get




Taking the term (x + a + b + c) common from C1, we get



Applying R2→ R2 – R1, we get




Applying R3→ R3 – R1, we get




Expanding the determinant along C1, we have


Δ = (x + a + b + c)(1)[(x)(x) – (0)(0)]


⇒ Δ = (x + a + b + c)(x)(x)


∴ Δ = x2(x + a + b + c)


The given equation is Δ = 0.


⇒ x2(x + a + b + c) = 0


Case – I:


x2 = 0 ⇒ x = 0


Case – II:


x + a + b + c = 0 ⇒ x = –(a + b + c)


Thus, 0 and –(a + b + c) are the roots of the given determinant equation.



Question 75.

Solve the following determinant equations:



Answer:


Let


We need to find the roots of Δ = 0.


Recall that the value of a determinant remains same if we apply the operation Ri→ Ri + kRj or Ci→ Ci + kCj.


Applying C1→ C1 + C2, we get




Applying C1→ C1 + C3, we get




Taking the term (3x + a) common from C1, we get



Applying R2→ R2 – R1, we get




Applying R3→ R3 – R1, we get




Expanding the determinant along C1, we have


Δ = (3x + a)(1)[(a)(a) – (0)(0)]


⇒ Δ = (3x + a)(a)(a)


∴ Δ = a2(3x + a)


The given equation is Δ = 0.


⇒ a2(3x + a) = 0


However, a ≠ 0 according to the given condition.


⇒ 3x + a = 0


⇒ 3x = –a



Thus, is the root of the given determinant equation.



Question 76.

Solve the following determinant equations:



Answer:


Let


We need to find the roots of Δ = 0.


Recall that the value of a determinant remains same if we apply the operation Ri→ Ri + kRj or Ci→ Ci + kCj.


Applying C1→ C1 + C2, we get




Applying C1→ C1 + C3, we get




Taking the term (3x – 2) common from C1, we get



Applying R2→ R2 – R1, we get




Applying R3→ R3 – R1, we get




Expanding the determinant along C1, we have


Δ = (3x – 2)(1)[(3x – 11)(3x – 11) – (0)(0)]


⇒ Δ = (3x – 2)(3x – 11)(3x – 11)


∴ Δ = (3x – 2)(3x – 11)2


The given equation is Δ = 0.


⇒ (3x – 2)(3x – 11)2 = 0


Case – I:


3x – 2 = 0


⇒ 3x = 2



Case – II:


(3x – 11)2 = 0


⇒ 3x – 11 = 0


⇒ 3x = 11



Thus, and are the roots of the given determinant equation.



Question 77.

Solve the following determinant equations:



Answer:


Let


We need to find the roots of Δ = 0.


Recall that the value of a determinant remains same if we apply the operation Ri→ Ri + kRj or Ci→ Ci + kCj.


Applying R2→ R2 – R1, we get




Applying R3→ R3 – R1, we get





Taking (a – x) and (b – x) common from R2 and R3, we get



Expanding the determinant along C1, we have


Δ = (a – x)(b – x)(1)[(1)(b + x) – (1)(a + x)]


⇒ Δ = (a – x)(b – x)[b + x – a – x]


∴ Δ = (a – x)(b – x)(b – a)


The given equation is Δ = 0.


⇒ (a – x)(b – x)(b – a) = 0


However, a ≠ b according to the given condition.


⇒ (a – x)(b – x) = 0


Case – I:


a – x = 0 ⇒ x = a


Case – II:


b – x = 0 ⇒ x = b


Thus, a and b are the roots of the given determinant equation.



Question 78.

Solve the following determinant equations:



Answer:


Let


We need to find the roots of Δ = 0.


Recall that the value of a determinant remains same if we apply the operation Ri→ Ri + kRj or Ci→ Ci + kCj.


Applying C1→ C1 + C2, we get




Applying C1→ C1 + C3, we get




Taking the term (x + 9) common from C1, we get



Applying R2→ R2 – R1, we get




Applying R3→ R3 – R1, we get




Expanding the determinant along C1, we have


Δ = (x + 9)(1)[(x – 1)(x – 1) – (0)(0)]


⇒ Δ = (x + 9)(x – 1)(x – 1)


∴ Δ = (x + 9)(x – 1)2


The given equation is Δ = 0.


⇒ x2(x + a + b + c) = 0


Case – I:


x+ 9 = 0 ⇒ x = –9


Case – II:


(x – 1)2 = 0


⇒ x – 1 = 0


∴ x = 1


Thus, –9 and 1 are the roots of the given determinant equation.



Question 79.

Solve the following determinant equations:



Answer:


Let


We need to find the roots of Δ = 0.


Recall that the value of a determinant remains same if we apply the operation Ri→ Ri + kRj or Ci→ Ci + kCj.


Applying R2→ R2 – R1, we get




Applying R3→ R3 – R1, we get





Taking (b – x) and (c – x) common from R2 and R3, we get



Expanding the determinant along C1, we have


Δ = (b – x)(c – x)(1)[(1)(c2 + cx + x2) – (1)(b2 + bx + x2)]


⇒ Δ = (b – x)(c – x)[c2 + cx + x2 – b2 – bx – x2]


⇒ Δ = (b – x)(c – x)[c2 – b2 + cx – bx]


⇒ Δ = (b – x)(c – x)[(c – b)(c + b) + (c – b)x]


∴ Δ = (b – x)(c – x)(c – b)(c + b + x)


The given equation is Δ = 0.


⇒ (b – x)(c – x)(c – b)(c + b + x) = 0


However, b ≠ c according to the given condition.


⇒ (b – x)(c – x)(c + b + x) = 0


Case – I:


b – x = 0 ⇒ x = b


Case – II:


c – x = 0 ⇒ x = c


Case – III:


c + b + x = 0 ⇒ x = –(b + c)


Thus, b, c and –(b + c) are the roots of the given determinant equation.



Question 80.

Solve the following determinant equations:



Answer:


Let


We need to find the roots of Δ = 0.


Recall that the value of a determinant remains same if we apply the operation Ri→ Ri + kRj or Ci→ Ci + kCj.


Applying R2→ R2 – R3, we get




Applying C2→ C2 – C1, we get




Applying C3→ C3 – C1, we get




Expanding the determinant along R2, we have


Δ = – (1)[(–4 – x)(3) – (6)(x – 8)]


⇒ Δ = – [–12 – 3x – 6x + 48]


⇒ Δ = – [– 9x + 36]


∴ Δ = 9x – 36


The given equation is Δ = 0.


⇒ 9x – 36 = 0


⇒ 9x = 36


∴ x = 4


Thus, 4 is the root of the given determinant equation.



Question 81.

Solve the following determinant equations:



Answer:


Let


We need to find the roots of Δ = 0.


Recall that the value of a determinant remains same if we apply the operation Ri→ Ri + kRj or Ci→ Ci + kCj.


Applying R2→ R2 – R1, we get




Taking the term p common from R2, we get



Applying C1→ C1 – C2, we get




Expanding the determinant along C1, we have


Δ = p(2 – x)[(1)(1) – (1)(x)]


∴ Δ = p(2 – x)(1 – x)


The given equation is Δ = 0.


⇒ p(2 – x)(1 – x) = 0


Assuming p ≠ 0, we get


⇒ (2 – x)(1 – x) = 0


Case – I:


2 – x = 0 ⇒ x = 2


Case – II:


1 – x = 0 ⇒ x = 1


Thus, 1 and 2 are the roots of the given determinant equation.



Question 82.

Solve the following determinant equations:



Answer:


Let


We need to find the roots of Δ = 0.


Recall that the value of a determinant remains same if we apply the operation Ri→ Ri + kRj or Ci→ Ci + kCj.


Applying C1→ C1 + C2, we get




Applying R2→ R2 – R1, we get




Applying R3→ R3 – 3R1, we get




Expanding the determinant along C1, we have


Δ = (1)[(10)(2 – 3sin(3θ)) – (20)(cos(2θ) – sin(3θ))]


⇒ Δ = [20 – 30sin(3θ) – 20cos(2θ) + 20sin(3θ)]


⇒ Δ = 20 – 10sin(3θ) – 20cos(2θ)


From trigonometry, we have sin(3θ) = 3sinθ – 4sin3θ and cos(2θ) = 1 – 2sin2θ.


⇒ Δ = 20 – 10(3sinθ – 4sin3θ) – 20(1 – 2sin2θ)


⇒ Δ = 20 – 30sinθ + 40sin3θ – 20 + 40sin2θ


⇒ Δ = –30sinθ + 40sin2θ + 40sin3θ


∴ Δ = 10(sinθ)(–3 + 4sinθ + 4sin2θ)


The given equation is Δ = 0.


⇒ 10(sinθ)(–3 + 4sinθ + 4sin2θ) = 0


⇒ (sinθ)(–3 + 4sinθ + 4sin2θ) = 0


Case – I:


sin θ = 0 ⇒ θ = kπ, where k ϵ Z


Case – II:


–3 + 4sinθ + 4sin2θ = 0


⇒ 4sin2θ + 4sinθ – 3 = 0


⇒ 4sin2θ + 6sinθ – 2sinθ – 3 = 0


⇒ 2sinθ(2sinθ + 3) – 1(2sinθ + 3) = 0


⇒ (2sinθ – 1)(2sinθ + 3) = 0


⇒ 2sinθ – 1 = 0 or 2sinθ + 3 = 0


⇒ 2sinθ = 1 or 2sinθ = –3


or


However, as –1 ≤ sin θ ≤ 1.



, where k ϵ Z


Thus, kπ and for all integral values of k are the roots of the given determinant equation.



Question 83.

If a, b and c are all non-zero and , then prove that .


Answer:

Let


Given that Δ = 0.


We can write the determinant Δ as



Taking a, b and c common from C1, C2 and C3, we get



Recall that the value of a determinant remains same if we apply the operation Ri→ Ri + kRj or Ci→ Ci + kCj.


Applying C1→ C1 + C2, we get




Applying C1→ C1 + C3, we get




Taking common from C1, we get



Applying R2→ R2 – R1, we get




Applying R3→ R3 – R1, we get




Expanding the determinant along C1, we have




We have Δ = 0.



It is given that a, b and c are all non-zero.



Thus, when and a, b, c are all non-zero.



Question 84.

If , then using properties of determinants, find the value of , where .


Answer:

Let


Given that Δ = 0.


Recall that the value of a determinant remains same if we apply the operation Ri→ Ri + kRj or Ci→ Ci + kCj.


Applying R2→ R2 – R1, we get




Applying R3→ R3 – R1, we get




Expanding the determinant along C3, we have


⇒ Δ = (c – z)[0 – (–x)(y)] – 0 + z[(a)(y) – (–x)(b – y)]


⇒ Δ = (c – z)(xy) + z[ay + xb – xy]


⇒ Δ = cxy – xyz + ayz + bxz – xyz


∴ Δ = ayz + bxz + cxy – 2xyz


We have Δ = 0


⇒ ayz + bxz + cxy – 2xyz = 0


⇒ ayz + bxz + cxy = 2xyz





Thus, when.




Exercise 6.3
Question 1.

Find the area of the triangle with vertices at the points:

(3, 8), (– 4, 2) and (5, – 1)


Answer:

Given: – Vertices of the triangle:


(3, 8), (– 4, 2) and (5, – 1)


We know that,


If vertices of a triangle are (x1,y1), (x2,y2) and (x3,y3), then the area of the triangle is given by:



Now, substituting given value in above formula



Expanding along R1





sq.units


Thus area of triangle is sq.units



Question 2.

Find the area of the triangle with vertices at the points:

(2, 7) (1, 1) and (10, 8)


Answer:

Given: – Vertices of the triangle:


(2, 7) (1, 1) and (10, 8)


We know that,


If vertices of a triangle are (x1,y1), (x2,y2) and (x3,y3), then the area of the triangle is given by:



Now, substituting given value in above formula



Expanding along R1





sq.units


Thus area of triangle is sq.units



Question 3.

Find the area of the triangle with vertices at the points:

(– 1, – 8), (– 2, – 3) and (3, 2)


Answer:

Given: – Vertices of the triangle:


(– 1, – 8), (– 2, – 3) and (3, 2)


We know that,


If vertices of a triangle are (x1,y1), (x2,y2) and (x3,y3), then the area of the triangle is given by:



Now, substituting given value in above formula



Expanding along R1





sq.units


as area cannot be negative


Therefore, 15 sq.unit is the area


Thus area of triangle is 15 sq.units



Question 4.

Find the area of the triangle with vertices at the points:

(0, 0) (6, 0) and (4, 3)


Answer:

Given: – Vertices of the triangle:


(0, 0) (6, 0) and (4, 3)


We know that,


If vertices of a triangle are (x1,y1), (x2,y2) and (x3,y3), then the area of the triangle is given by:



Now, substituting given value in above formula



Expanding along R1





= 9 sq.units


Thus area of triangle is 9 sq.units



Question 5.

Using determinants show that the following points are collinear:

(1, – 1), (2, 1) and (4, 5)


Answer:

Given: – (1, – 1), (2, 1) and (4, 5) are three points

Tip: – For Three points to be collinear, the area of the triangle formed by these points will be zero


Now, we know that,


vertices of a triangle are (x1,y1), (x2,y2) and (x3,y3), then the area of the triangle is given by:



Now,


Substituting given value in above formula



R.H.S



Expanding along R1





= 0


= LHS


Since, Area of triangle is zero.


Hence, points are collinear.



Question 6.

Using determinants show that the following points are collinear:

(3, – 2), (8, 8) and (5, 2)


Answer:

Given: – (3, – 2), (8, 8) and (5, 2) are three points

Tip: – For Three points to be collinear, the area of triangle formed by these points will be zero


Now, we know that,


vertices of a triangle are (x1,y1), (x2,y2) and (x3,y3), then the area of the triangle is given by:



Now,


Substituting given value in above formula



R.H.S



Expanding along R1





= 0


= LHS


Since, Area of triangle is zero


Hence, points are collinear.



Question 7.

Using determinants show that the following points are collinear:

(2, 3), (– 1, – 2) and (5, 8)


Answer:

Given: – (2, 3), (– 1, – 2) and (5, 8) are three points


Tip: – For Three points to be collinear, the area of the triangle formed by these points will be zero


Now, we know that,


vertices of a triangle are (x1,y1), (x2,y2) and (x3,y3), then the area of the triangle is given by:



Now,


Substituting given value in above formula



R.H.S



Expanding along R1





= 0


= LHS


Since, Area of triangle is zero


Hence, points are collinear.



Question 8.

If the points (a, 0), (o, b) and (1, 1) are collinear, prove that a + b = ab.


Answer:

Given: – (a, 0), (o, b) and (1, 1) are collinear points

To Prove: – a + b = ab


Proof: –


Tip: – If Three points to be collinear, then the area of the triangle formed by these points will be zero


Now, we know that,


vertices of a triangle are (x1,y1), (x2,y2) and (x3,y3), then the area of the triangle is given by:



Thus



Expanding along R1





⇒ a + b = ab


Hence Proved



Question 9.

Using determinants prove that the points (a, b) (a’, b’) and (a – a’, b – b’) are collinear if ab’ = a’b.


Answer:

Given: – (a, b) (a’, b’) and (a – a’, b – b’) are points and ab’ = a’b


To Prove: – (a, b) (a’, b’) and (a – a’, b – b’) are collinear points


Proof: –


Tip: – If three points to be collinear, then the area of the triangle formed by these points will be zero.


Now, we know that,


vertices of a triangle are (x1,y1), (x2,y2) and (x3,y3), then the area of the triangle is given by:



Thus



Expanding along R1





⇒ ab – ab = 0


⇒ ab = ab


Hence, Proved.



Question 10.

Find the value of λ so that the points (1, – 5), (– 4, 5) and (λ, 7) are collinear.


Answer:

Given: – (1, – 5), (– 4, 5) and (λ, 7) are collinear


Tip: – For Three points to be collinear, the area of the triangle formed by these points will be zero


Now, we know that,


vertices of a triangle are (x1,y1), (x2,y2) and (x3,y3), then the area of the triangle is given by:



Now,


Substituting given value in above formula



Expanding along R1





50 – 10λ = 0


⇒ λ = – 5




Question 11.

Find the value of x if the area of a triangle is 35 square cms with vertices (x, 4), (2, – 6) and (5, 4).


Answer:

Given: – Vertices of triangle are (x, 4), (2, – 6) and (5, 4) and area of triangle is 35 sq.cms


Tip: – If vertices of a triangle are (x1,y1), (x2,y2) and (x3,y3), then the area of the triangle is given by:



Now,


Substituting given value in above formula



Removing modulus



Expanding along R1



⇒ [x(– 10) – 4(– 3) + 1(8 – 30)] = ± 70


⇒ [ – 10x + 12 + 38] = ± 70


⇒ ±70 = – 10x + 50


Taking + ve sign, we get


⇒ + 70 = – 10x + 50


⇒ 10x = – 20


⇒ x = – 2


Taking – ve sign, we get


⇒ – 70 = – 10x + 50


⇒ 10x = 120


⇒ x = 12


Thus x = – 2, 12



Question 12.

Using determinants, find the area of a triangle whose vertices are (1, 4), (2, 3) and (– 5, – 3). Are the given points collinear?


Answer:

Given: – Vertices are (1, 4), (2, 3) and (– 5, – 3)


Tip: – If vertices of a triangle are (x1,y1), (x2,y2) and (x3,y3), then the area of the triangle is given by:



Now,


Substituting given value in above formula



Expanding along R1





Since area can’t be negative



Tip: – For Three points to be collinear, the area of the triangle formed by these points will be zero


Now, as the area is not zero


Therefore, Points (1, 4), (2, 3) and (– 5, – 3) are not collinear.



Question 13.

Using determinants, find the area of the triangle with vertices (– 3, 5), (3, – 6) and (7, 2).


Answer:

Given: – Vertices are (– 3, 5), (3, – 6) and (7, 2)


Tip: – If vertices of a triangle are (x1,y1), (x2,y2) and (x3,y3), then the area of the triangle is given by:



Now,


Substituting given value in above formula



Expanding along R1






⇒ Δ = 46 sq. units



Question 14.

Using determinants, find the value of k so that the points (k, 2 – 2 k), (– k + 1, 2k) and (– 4 – k, 6 – 2k) may be collinear.


Answer:

Given: – Points are (k, 2 – 2 k), (– k + 1, 2k) and (– 4 – k, 6 – 2k) which are collinear


Tip: – For Three points to be collinear, the area of the triangle formed by these points will be zero.


If vertices of a triangle are (x1,y1), (x2,y2) and (x3,y3), then the area of the triangle is given by:



Now,


Substituting given value in above formula



Expanding along R1



⇒ k(2k – 6 + 2k) – (2 – 2k)(– k + 1 + 4 + k) + 1(6 – 2k – 6k + 2k2 + 8k + 2k2) = 0


⇒ 4k2 – 6k – 10 + 10k + 6 + 4k2 = 0


⇒ 8k2 + 4k – 4 = 0


⇒ 8k2 + 8k – 4k – 4 = 0


⇒ 8k(k + 1) – 4(k + 1) = 0


⇒ (8k – 4)(k + 1) = 0


If 8k – 4 = 0



And, If k + 1 = 0


⇒ K = – 1


Hence, k = – 1, 0.5



Question 15.

If the points (x, 2), (5, – 2) and (8, 8) are collinear, find x using determinants.


Answer:

Given: – (x, 2), (5, – 2) and (8, 8) are collinear points


Tip: – For Three points to be collinear, the area of the triangle formed by these points will be zero.


Now, we know that,


Vertices of a triangle are (x1,y1), (x2,y2) and (x3,y3), then the area of the triangle is given by:



Now,


Substituting given value in above formula




Expanding along R1



⇒ [x(– 6) + 2(– 3) + 1(24)] = 0


⇒ – 6x – 6 + 24 = 0


⇒ x = 3



Question 16.

If the points (3, – 2), (x,2) and (8,8) are collinear, find x using determinant.


Answer:

Given: – (3, – 2), (x,2) and (8,8) are collinear points


Tip: – For Three points to be collinear, the area of the triangle formed by these points will be zero


Now, we know that,


Vertices of a triangle are (x1,y1), (x2,y2) and (x3,y3), then the area of the triangle is given by:



Now,


Substituting given value in above formula




Expanding along R1



⇒ [x(– 6) + 2(x – 8) + 1(8x – 16)] = 0


⇒ – 6x + 2x – 16 + 8x – 16 = 0


⇒ 10x = 50


⇒ x = 5



Question 17.

Using determinants, find the equation of the line joining the points

(1, 2) and (3, 6)


Answer:

Given: – (1, 2) and (3, 6) are collinear points


Tip: – For Three points to be collinear, the area of the triangle formed by these points will be zero


Now, we know that,


Vertices of a triangle are (x1,y1), (x2,y2) and (x3,y3), then the area of the triangle is given by:



Now,


Let, 3rd point be (x,y)


Substituting given value in above formula




Expanding along R1



⇒ [x(– 4) – y(– 2) + 1(0)] = 0


⇒ – 4x + 2y = 0


⇒ y = 2x


It’s the equation of line



Question 18.

Using determinants, find the equation of the line joining the points

(3, 1) and (9, 3)


Answer:

Given: – (3, 1) and (9, 3) are collinear points


Tip: – For Three points to be collinear, the area of triangle formed by these points will be zero


Now, we know that,


Vertices of a triangle are (x1,y1), (x2,y2) and (x3,y3), then the area of the triangle is given by:



Now,


Let, 3rd point be (x,y)


Substituting given value in above formula




Expanding along R1



⇒ [x(– 2) – y(– 6) + 1(0)] = 0


⇒ – 2x + 6y = 0


⇒ x – 3y = 0


It’s the equation of line



Question 19.

Find values of K, if the area of a triangle is 4 square units whose vertices are

(k,0), (4,0) and (0,2)


Answer:

Given: – Vertices of triangle are (k, 0), (4, 0) and (0, 2) and area of triangle is 4 sq. units


Tip: – If vertices of a triangle are (x1,y1), (x2,y2) and (x3,y3), then the area of the triangle is given by:



Now,


Substituting given value in above formula



Removing modulus



Expanding along R1



⇒ [k(– 2) – 0(4) + 1(8 – 0)] = ±8


⇒ [ – 2k + 8] = ± 8


Taking + ve sign, we get


⇒ + 8 = – 2x + 8


⇒ – 2k = 0


⇒ k = 0


Taking – ve sign, we get


⇒ – 8 = – 2x + 8


⇒ – 2x = – 16


⇒ x = 8


Thus x = 0, 8



Question 20.

Find values of K, if the area of a triangle is 4 square units whose vertices are

(– 2,0), (0, 4) and (0, k)


Answer:

Given: – Vertices of triangle are (– 2,0), (0, 4) and (0, k) and the area of the triangle is 4 sq. units.


Tip: – If vertices of a triangle are (x1,y1), (x2,y2) and (x3,y3), then the area of the triangle is given by:



Now,


Substituting given value in above formula



Removing modulus



Expanding along R1



⇒ [ – 2(4 – k) – 0(0) + 1(0 – 0)] = ±8


⇒ – 8 + 2k = ± 8


Taking + ve sign, we get


⇒ 8 = – 8 + 2k


⇒ 2k = 16


⇒ k = 8


Taking – ve sign, we get


⇒ – 8 = 2x – 8


⇒ 2k = 0


⇒ k = 0


Thus k = 0, 8




Exercise 6.4
Question 1.

Solve the following systems of linear equations by Cramer’s rule:

x – 2y = 4

– 3x + 5y = – 7


Answer:

Given: - Two equations x – 2y = 4 and – 3x + 5y = – 7


Tip: - Theorem - Cramer’s Rule


Let there be a system of n simultaneous linear equations and with n unknown given by







and let Dj be the determinant obtained from D after replacing the jth column by



Then,


provided that D ≠ 0


Now, here we have


x – 2y = 4


– 3x + 5y = – 7


So by comparing with the theorem, let's find D, D1 and D2



Solving determinant, expanding along 1st row


⇒ D = 5(1) – ( – 3)( – 2)


⇒ D = 5 – 6


⇒ D = – 1


Again,



Solving determinant, expanding along 1st row


⇒ D1 = 5(4) – ( – 7)( – 2)


⇒ D1 = 20 – 14


⇒ D1 = 6


and



Solving determinant, expanding along 1st row


⇒ D2 = 1( – 7) – ( – 3)(4)


⇒ D2 = – 7 + 12


⇒ D2 = 5


Thus by Cramer’s Rule, we have




⇒ x = – 6


and




⇒ y = – 5



Question 2.

Solve the following systems of linear equations by Cramer’s rule:

2x – y = 1

7x – 2y = – 7


Answer:

Given: - Two equations 2x – y = 1 and 7x – 2y = – 7


Tip: - Theorem – Cramer’s Rule


Let there be a system of n simultaneous linear equations and with n unknown given by







and let Dj be the determinant obtained from D after replacing the jth column by



Then,


provided that D ≠ 0


Now, here we have


2x – y = 1


7x – 2y = – 7


So by comparing with the theorem, let's find D, D1 and D2



Solving determinant, expanding along 1st row


⇒ D = 2( – 2) – (7)( – 1)


⇒ D = – 4 + 7


⇒ D = 3


Again,



Solving determinant, expanding along 1st row


⇒ D1 = 1( – 2) – ( – 7)( – 1)


⇒ D1 = – 2 – 7


⇒ D1 = – 9


and



Solving determinant, expanding along 1st row


⇒ D2 = 2( – 7) – (7)(1)


⇒ D2 = – 14 – 7


⇒ D2 = – 21


Thus by Cramer’s Rule, we have




⇒ x = – 3


and




⇒ y = – 7



Question 3.

Solve the following systems of linear equations by Cramer’s rule:

2x – y = 17

3x + 5y = 6


Answer:

Given: - Two equations 2x – y = 17 and 3x + 5y = 6


Tip: - Theorem – Cramer’s Rule


Let there be a system of n simultaneous linear equations and with n unknown given by







and let Dj be the determinant obtained from D after replacing the jth column by



Then,


provided that D ≠ 0


Now, here we have


2x – y = 17


3x + 5y = 6


So by comparing with the theorem, let's find D, D1 and D2



Solving determinant, expanding along 1st row


⇒ D = 2(5) – (3)( – 1)


⇒ D = 10 + 3


⇒ D = 13


Again,



Solving determinant, expanding along 1st row


⇒ D1 = 17(5) – (6)( – 1)


⇒ D1 = 85 + 6


⇒ D1 = 91


and



Solving determinant, expanding along 1st row


⇒ D2 = 2(6) – (17)(3)


⇒ D2 = 12 – 51


⇒ D2 = – 39


Thus by Cramer’s Rule, we have




⇒ x = 7


and




⇒ y = – 3



Question 4.

Solve the following systems of linear equations by Cramer’s rule:

3x + y = 19

3x – y = 23


Answer:

Given: - Two equations 3x + y = 19 and 3x – y = 23


Tip: - Theorem – Cramer’s Rule


Let there be a system of n simultaneous linear equations and with n unknown given by







and let Dj be the determinant obtained from D after replacing the jth column by



Then,


provided that D ≠ 0


Now, here we have


3x + y = 19


3x – y = 23


So by comparing with the theorem, let's find D, D1 and D2



Solving determinant, expanding along 1st row


⇒ D = 3( – 1) – (3)(1)


⇒ D = – 3 – 3


⇒ D = – 6


Again,



Solving determinant, expanding along 1st row


⇒ D1 = 19( – 1) – (23)(1)


⇒ D1 = – 19 – 23


⇒ D1 = – 42


and



Solving determinant, expanding along 1st row


⇒ D2 = 3(23) – (19)(3)


⇒ D2 = 69 – 57


⇒ D2 = 12


Thus by Cramer’s Rule, we have




⇒ x = 7


and




⇒ y = – 2



Question 5.

Solve the following systems of linear equations by Cramer’s rule:

2x – y = – 2

3x + 4y = 3


Answer:

Given : – Two equations 2x – y = – 2 and 3x + 4y = 3


Tip: - Theorem – Cramer’s Rule


Let there be a system of n simultaneous linear equations and with n unknown given by







and let Dj be the determinant obtained from D after replacing the jth column by



Then,


provided that D ≠ 0


Now, here we have


2x – y = – 2


3x + 4y = 3


So by comparing with the theorem, let's find D, D1 and D2



Solving determinant, expanding along 1st row


⇒ D = 2(4) – (3)( – 1)


⇒ D = 8 + 3


⇒ D = 11


Again,



Solving determinant, expanding along 1st row


⇒ D1 = – 2(4) – (3)( – 1)


⇒ D1 = – 8 + 3


⇒ D1 = – 5


and



Solving determinant, expanding along 1st row


⇒ D2 = 3(2) – ( – 2)(3)


⇒ D2 = 6 + 6


⇒ D2 = 12


Thus by Cramer’s Rule, we have




and





Question 6.

Solve the following systems of linear equations by Cramer’s rule:

3x + ay = 4

2x + ay = 2, a≠0


Answer:

Given: - Two equations 3x + ay = 4 and 2x + ay = 2, a ≠ 0


Tip: - Theorem – Cramer’s Rule


Let there be a system of n simultaneous linear equations and with n unknown given by







and let Dj be the determinant obtained from D after replacing the jth column by



Then,


provided that D ≠ 0


Now, here we have


3x + ay = 4


2x + ay = 2, a≠0


So by comparing with the theorem, let's find D, D1 and D2



Solving determinant, expanding along 1st row


⇒ D = 3(a) – (2)(a)


⇒ D = 3a – 2a


⇒ D = a


Again,



Solving determinant, expanding along 1st row


⇒ D1 = 4(a) – (2)(a)


⇒ D = 4a – 2a


⇒ D = 2a


and



Solving determinant, expanding along 1st row


⇒ D2 = 3(2) – (2)(4)


⇒ D = 6 – 8


⇒ D = – 2


Thus by Cramer’s Rule, we have




⇒ x = 2


and





Question 7.

Solve the following systems of linear equations by Cramer’s rule:

2x + 3y = 10

x + 6y = 4


Answer:

Given: - Two equations 2x – 3y = 10 and x + 6y = 4


Tip: - Theorem – Cramer’s Rule


Let there be a system of n simultaneous linear equations and with n unknown given by







and let Dj be the determinant obtained from D after replacing the jth column by



Then,


provided that D ≠ 0


Now, here we have


2x + 3y = 10


x + 6y = 4


So by comparing with the theorem, let's find D, D1 and D2



Solving determinant, expanding along 1st row


⇒ D = 2(6) – (3)(1)


⇒ D = 12 – 3


⇒ D = 9


Again,



Solving determinant, expanding along 1st row


⇒ D1 = 10(6) – (3)(4)


⇒ D = 60 – 12


⇒ D = 48


and



Solving determinant, expanding along 1st row


⇒ D2 = 2(4) – (10)(1)


⇒ D2 = 8 – 10


⇒ D2 = – 2


Thus by Cramer’s Rule, we have





and






Question 8.

Solve the following systems of linear equations by Cramer’s rule:

5x + 7y = – 2

4x + 6y = – 3


Answer:

Given: - Two equations 5x + 7y = – 2 and 4x + 6y = – 3


Tip: - Theorem – Cramer’s Rule


Let there be a system of n simultaneous linear equations and with n unknown given by







and let Dj be the determinant obtained from D after replacing the jth column by



Then,


provided that D ≠ 0


Now, here we have


5x + 7y = – 2


4x + 6y = – 3


So by comparing with the theorem, let's find D, D1 and D2



Solving determinant, expanding along 1st row


⇒ D = 5(6) – (7)(4)


⇒ D = 30 – 28


⇒ D = 2


Again,



Solving determinant, expanding along 1st row


⇒ D1 = – 2(6) – (7)( – 3)


⇒ D1 = – 12 + 21


⇒ D1 = 9


and



Solving determinant, expanding along 1st row


⇒ D2 = – 3(5) – ( – 2)(4)


⇒ D2 = – 15 + 8


⇒ D2 = – 7


Thus by Cramer’s Rule, we have





and






Question 9.

Solve the following systems of linear equations by Cramer’s rule:

9x + 5y = 10

3y – 2x = 8


Answer:

Given: - Two equations 9x + 5y = 10 and 3y – 2x = 8


Tip: - Theorem – Cramer’s Rule


Let there be a system of n simultaneous linear equations and with n unknown given by







and let Dj be the determinant obtained from D after replacing the jth column by



Then,


provided that D ≠ 0


Now, here we have


9x + 5y = 10


3y – 2x = 8


So by comparing with the theorem, let's find D, D1 and D2



Solving determinant, expanding along 1st row


⇒ D = 3(9) – (5)( – 2)


⇒ D = 27 + 10


⇒ D = 37


Again,



Solving determinant, expanding along 1st row


⇒ D1 = 10(3) – (8)(5)


⇒ D1 = 30 – 40


⇒ D1 = – 10


and



Solving determinant, expanding along 1st row


⇒ D2 = 9(8) – (10)( – 2)


⇒ D2 = 72 + 20


⇒ D2 = 92


Thus by Cramer’s Rule, we have





and






Question 10.

Solve the following systems of linear equations by Cramer’s rule:

x + 2y = 1

3x + y = 4


Answer:

Given: - Two equations x + 2y = 1 and 3x + y = 4


Tip: - Theorem – Cramer’s Rule


Let there be a system of n simultaneous linear equations and with n unknown given by







and let Dj be the determinant obtained from D after replacing the jth column by



Then,


provided that D ≠ 0


Now, here we have


x + 2y = 1


3x + y = 4


So by comparing with theorem, lets find D, D1 and D2



Solving determinant, expanding along 1st row


⇒ D = 1(1) – (3)(2)


⇒ D = 1 – 6


⇒ D = – 5


Again,



Solving determinant, expanding along 1st row


⇒ D1 = 1(1) – (2)(4)


⇒ D1 = 1 – 8


⇒ D1 = – 7


and



Solving determinant, expanding along 1st row


⇒ D2 = 1(4) – (1)(3)


⇒ D2 = 4 – 3


⇒ D2 = 1


Thus by Cramer’s Rule, we have





and






Question 11.

Solve the following system of the linear equations by Cramer’s rule:

3x + y + z = 2

2x – 4y + 3z = – 1

4x + y – 3z = – 11


Answer:

Given: - Equations are: –


3x + y + z = 2


2x – 4y + 3z = – 1


4x + y – 3z = – 11


Tip: - Theorem – Cramer’s Rule


Let there be a system of n simultaneous linear equations and with n unknown given by







and let Dj be the determinant obtained from D after replacing the jth column by



Then,


provided that D ≠ 0


Now, here we have


3x + y + z = 2


2x – 4y + 3z = – 1


4x + y – 3z = – 11


So by comparing with the theorem, let's find D, D1, D2 and D3



Solving determinant, expanding along 1st row


⇒ D = 3[( – 4)( – 3) – (3)(1)] – 1[(2)( – 3) – 12] + 1[2 – 4( – 4)]


⇒ D = 3[12 – 3] – [ – 6 – 12] + [2 + 16]


⇒ D = 27 + 18 + 18


⇒ D = 63


Again,



Solving determinant, expanding along 1st row


⇒ D1 = 2[( – 4)( – 3) – (3)(1)] – 1[( – 1)( – 3) – ( – 11)(3)] + 1[( – 1) – ( – 4)( – 11)]


⇒ D1 = 2[12 – 3] – 1[3 + 33] + 1[ – 1 – 44]


⇒ D1 = 2[9] – 36 – 45


⇒ D1 = 18 – 36 – 45


⇒ D1 = – 63


Again



Solving determinant, expanding along 1st row


⇒ D2 = 3[3 + 33] – 2[ – 6 – 12] + 1[ – 22 + 4]


⇒ D2 = 3[36] – 2( – 18) – 18


⇒ D2 = 126


And,



Solving determinant, expanding along 1st row


⇒ D3 = 3[44 + 1] – 1[ – 22 + 4] + 2[2 + 16]


⇒ D3 = 3[45] – 1( – 18) + 2(18)


⇒ D3 = 135 + 18 + 36


⇒ D3 = 189


Thus by Cramer’s Rule, we have




⇒ x = – 1


again,




⇒ y = 2


and,




⇒ z = 3



Question 12.

Solve the following system of the linear equations by Cramer’s rule:

x – 4y – z = 11

2x – 5y + 2z = 39

– 3x + 2y + z = 1


Answer:

Given: - Equations are: –


x – 4y – z = 11


2x – 5y + 2z = 39


– 3x + 2y + z = 1


Tip: - Theorem – Cramer’s Rule


Let there be a system of n simultaneous linear equations and with n unknown given by







and let Dj be the determinant obtained from D after replacing the jth column by



Then,


provided that D ≠ 0


Now, here we have


x – 4y – z = 11


2x – 5y + 2z = 39


– 3x + 2y + z = 1


So by comparing with theorem, lets find D , D1 and D2



Solving determinant, expanding along 1st row


⇒ D = 1[( – 5)(1) – (2)(2)] + 4[(2)(1) + 6] – 1[4 + 5( – 3)]


⇒ D = 1[ – 5 – 4] + 4[8] – [ – 11]


⇒ D = – 9 + 32 + 11


⇒ D = 34


Again,



Solving determinant, expanding along 1st row


⇒ D1 = 11[( – 5)(1) – (2)(2)] + 4[(39)(1) – (2)(1)] – 1[2(39) – ( – 5)(1)]


⇒ D1 = 11[ – 5 – 4] + 4[39 – 2] – 1[78 + 5]


⇒ D1 = 11[ – 9] + 4(37) – 83


⇒ D1 = – 99 – 148 – 45


⇒ D1 = – 34


Again



Solving determinant, expanding along 1st row


⇒ D2 = 1[39 – 2] – 11[2 + 6] – 1[2 + 117]


⇒ D2 = 1[37] – 11(8) – 119


⇒ D2 = – 170


And,



Solving determinant, expanding along 1st row


⇒ D3 = 1[ – 5 – (39)(2)] – ( – 4)[2 – (39)( – 3)] + 11[4 – ( – 5)( – 3)]


⇒ D3 = 1[ – 5 – 78] + 4(2 + 117) + 11(4 – 15)


⇒ D3 = – 83 + 4(119) + 11( – 11)


⇒ D3 = 272


Thus by Cramer’s Rule, we have




⇒ x = – 1


again,




⇒ y = – 5


and,




⇒ z = 8



Question 13.

Solve the following system of the linear equations by Cramer’s rule:

6x + y – 3z = 5

X + 3y – 2z = 5

2x + y + 4z = 8


Answer:

Given: - Equations are: –


6x + y – 3z = 5


X + 3y – 2z = 5


2x + y + 4z = 8


Tip: - Theorem – Cramer’s Rule


Let there be a system of n simultaneous linear equations and with n unknown given by







and let Dj be the determinant obtained from D after replacing the jth column by



Then,


provided that D ≠ 0


Now, here we have


6x + y – 3z = 5


x + 3y – 2z = 5


2x + y + 4z = 8


So by comparing with theorem, lets find D , D1 and D2



Solving determinant, expanding along 1st Row


⇒ D = 6[(4)(3) – (1)( – 2)] – 1[(4)(1) + 4] – 3[1 – 3(2)]


⇒ D = 6[12 + 2] – [8] – 3[ – 5]


⇒ D = 84 – 8 + 15


⇒ D = 91


Again, Solve D1 formed by replacing 1st column by B matrices


Here




Solving determinant, expanding along 1st Row


⇒ D1 = 5[(4)(3) – ( – 2)(1)] – 1[(5)(4) – ( – 2)(8)] – 3[(5) – (3)(8)]


⇒ D1 = 5[12 + 2] – 1[20 + 16] – 3[5 – 24]


⇒ D1 = 5[14] – 36 – 3( – 19)


⇒ D1 = 70 – 36 + 57


⇒ D1 = 91


Again, Solve D2 formed by replacing 1st column by B matrices


Here




Solving determinant


⇒ D2 = 6[20 + 16] – 5[4 – 2( – 2)] + ( – 3)[8 – 10]


⇒ D2 = 6[36] – 5(8) + ( – 3)( – 2)


⇒ D2 = 182


And, Solve D3 formed by replacing 1st column by B matrices


Here




Solving determinant, expanding along 1st Row


⇒ D3 = 6[24 – 5] – 1[8 – 10] + 5[1 – 6]


⇒ D3 = 6[19] – 1( – 2) + 5( – 5)


⇒ D3 = 114 + 2 – 25


⇒ D3 = 91


Thus by Cramer’s Rule, we have




⇒ x = 1


again,




⇒ y = 2


and,




⇒ z = 1



Question 14.

Solve the following system of the linear equations by Cramer’s rule:

x + y = 5

y + z = 3

x + z = 4


Answer:

Given: - Equations are: –


x + y = 5


y + z = 3


x + z = 4


Tip: - Theorem – Cramer’s Rule


Let there be a system of n simultaneous linear equations and with n unknown given by







and let Dj be the determinant obtained from D after replacing the jth column by



Then,


provided that D ≠ 0


Now, here we have


x + y = 5


y + z = 3


x + z = 4


So by comparing with theorem, lets find D , D1 and D2



Solving determinant, expanding along 1st Row


⇒ D = 1[1] – 1[ – 1] + 0[ – 1]


⇒ D = 1 + 1 + 0


⇒ D = 2


⇒ D = 2


Again, Solve D1 formed by replacing 1st column by B matrices


Here




Solving determinant, expanding along 1st Row


⇒ D1 = 5[1] – 1[(3)(1) – (4)(1)] + 0[0 – (4)(1)]


⇒ D1 = 5 – 1[3 – 4] + 0[– 4]


⇒ D1 = 5 – 1[ – 1] + 0


⇒ D1 = 5 + 1 + 0


⇒ D1 = 6


Again, Solve D2 formed by replacing 1st column by B matrices


Here




Solving determinant


⇒ D2 = 1[3 – 4] – 5[ – 1] + 0[0 – 3]


⇒ D2 = 1[ – 1] + 5 + 0


⇒ D2 = 4


And, Solve D3 formed by replacing 1st column by B matrices


Here




Solving determinant, expanding along 1st Row


⇒ D3 = 1[4 – 0] – 1[0 – 3] + 5[0 – 1]


⇒ D3 = 1[4] – 1( – 3) + 5( – 1)


⇒ D3 = 4 + 3 – 5


⇒ D3 = 2


Thus by Cramer’s Rule, we have




⇒ x = 3


again,




⇒ y = 2


and,




⇒ z = 1



Question 15.

Solve the following system of the linear equations by Cramer’s rule:

2y – 3z = 0

X + 3y = – 4

3x + 4y = 3


Answer:

Given: - Equations are: –


2y – 3z = 0


X + 3y = – 4


3x + 4y = 3


Tip: - Theorem – Cramer’s Rule


Let there be a system of n simultaneous linear equations and with n unknown given by







and let Dj be the determinant obtained from D after replacing the jth column by



Then,


provided that D ≠ 0


Now, here we have


2y – 3z = 0


x + 3y = – 4


3x + 4y = 3


So by comparing with theorem, lets find D , D1 and D2



Solving determinant, expanding along 1st Row


⇒ D = 0[0] – 2[(0)(1) – 0] – 3[1(4) – 3(3)]


⇒ D = 0 – 0 – 3[4 – 9]


⇒ D = 0 – 0 + 15


⇒ D = 15


Again, Solve D1 formed by replacing 1st column by B matrices


Here




Solving determinant, expanding along 1st Row


⇒ D1 = 0[0] – 2[(0)( – 4) – 0] – 3[4( – 4) – 3(3)]


⇒ D1 = 0 – 0 – 3[ – 16 – 9]


⇒ D1 = 0 – 0 – 3( – 25)


⇒ D1 = 0 – 0 + 75


⇒ D1 = 75


Again, Solve D2 formed by replacing 2nd column by B matrices


Here




Solving determinant


⇒ D2 = 0[0] – 0[(0)(1) – 0] – 3[1(3) – 3( – 4)]


⇒ D2 = 0 – 0 + ( – 3)(3 + 12)


⇒ D2 = – 45


And, Solve D3 formed by replacing 3rd column by B matrices


Here




Solving determinant, expanding along 1st Row


⇒ D3 = 0[9 – ( – 4)4] – 2[(3)(1) – ( – 4)(3)] + 0[1(4) – 3(3)]


⇒ D3 = 0[25] – 2(3 + 12) + 0(4 – 9)


⇒ D3 = 0 – 30 + 0


⇒ D3 = – 30


Thus by Cramer’s Rule, we have




⇒ x = 5


again,




⇒ y = – 3


and,




⇒ z = – 2



Question 16.

Solve the following system of the linear equations by Cramer’s rule:

5x – 7y + z = 11

6x – 8y – z = 15

3x + 2y – 6z = 7


Answer:

Given: - Equations are: –


5x – 7y + z = 11


6x – 8y – z = 15


3x + 2y – 6z = 7


Tip: - Theorem – Cramer’s Rule


Let there be a system of n simultaneous linear equations and with n unknown given by







and let Dj be the determinant obtained from D after replacing the jth column by



Then,


provided that D ≠ 0


Now, here we have


5x – 7y + z = 11


6x – 8y – z = 15


3x + 2y – 6z = 7


So by comparing with theorem, lets find D , D1 and D2



Solving determinant, expanding along 1st Row


⇒ D = 5[( – 8)( – 6) – ( – 1)(2)] – 7[( – 6)(6) – 3( – 1)] + 1[2(6) – 3( – 8)]


⇒ D = 5[48 + 2] – 7[ – 36 + 3] + 1[12 + 24]


⇒ D = 250 – 231 + 36


⇒ D = 55


Again, Solve D1 formed by replacing 1st column by B matrices


Here




Solving determinant, expanding along 1st Row


⇒ D1 = 11[( – 8)( – 6) – (2)( – 1)] – ( – 7)[(15)( – 6) – ( – 1)(7)] + 1[(15)2 – (7)( – 8)]


⇒ D1 = 11[48 + 2] + 7[ – 90 + 7] + 1[30 + 56]


⇒ D1 = 11[50] + 7[ – 83] + 86


⇒ D1 = 550 – 581 + 86


⇒ D1 = 55


Again, Solve D2 formed by replacing 2nd column by B matrices


Here




Solving determinant, expanding along 1st Row


⇒ D2 = 5[(15)( – 6) – (7)( – 1)] – 11[(6)( – 6) – ( – 1)(3)] + 1[(6)7 – (15)(3)]


⇒ D2 = 5[ – 90 + 7] – 11[ – 36 + 3] + 1[42 – 45]


⇒ D2 = 5[ – 83] – 11( – 33) – 3


⇒ D2 = – 415 + 363 – 3


⇒ D2 = – 55


And, Solve D3 formed by replacing 3rd column by B matrices


Here




Solving determinant, expanding along 1st Row


⇒ D3 = 5[( – 8)(7) – (15)(2)] – ( – 7)[(6)(7) – (15)(3)] + 11[(6)2 – ( – 8)(3)]


⇒ D3 = 5[ – 56 – 30] – ( – 7)[42 – 45] + 11[12 + 24]


⇒ D3 = 5[ – 86] + 7[ – 3] + 11[36]


⇒ D3 = – 430 – 21 + 396


⇒ D3 = – 55


Thus by Cramer’s Rule, we have




⇒ x = 1


again,




⇒ y = – 1


and,




⇒ z = – 1



Question 17.

Solve the following system of the linear equations by Cramer’s rule:

2x – 3y – 4z = 29

– 2x + 5y – z = – 15

3x – y + 5z = – 11


Answer:

Given: - Equations are: –


2x – 3y – 4z = 29


– 2x + 5y – z = – 15


3x – y + 5z = – 11


Tip: - Theorem – Cramer’s Rule


Let there be a system of n simultaneous linear equations and with n unknown given by







and let Dj be the determinant obtained from D after replacing the jth column by



Then,


provided that D ≠ 0


Now, here we have


2x – 3y – 4z = 29


– 2x + 5y – z = – 15


3x – y + 5z = – 11


So by comparing with theorem, lets find D , D1 and D2



Solving determinant, expanding along 1st Row


⇒ D = 2[(5)(5) – ( – 1)( – 1)] – ( – 3)[( – 2)(5) – 3( – 1)] + ( – 4)[( – 2)( – 1) – 3(5)]


⇒ D = 2[25 – 1] + 3[ – 10 + 3] – 4[2 – 15]


⇒ D = 48 – 21 + 52


⇒ D = 79


Again, Solve D1 formed by replacing 1st column by B matrices


Here




Solving determinant, expanding along 1st Row


⇒ D1 = 29[(5)(5) – ( – 1)( – 1)] – ( – 3)[( – 15)(5) – ( – 11)( – 1)] + ( – 4)[( – 15)( – 1) – ( – 11)(5)]


⇒ D1 = 29[25 – 1] + 3[ – 75 – 11] – 4[15 + 55]


⇒ D1 = 29[24] + 3[ – 86] – 4(70)


⇒ D1 = 696 – 258 – 280


⇒ D1 = 158


Again, Solve D2 formed by replacing 2nd column by B matrices


Here




Solving determinant, expanding along 1st Row


⇒ D2 = 2[( – 15)(5) – ( – 11)( – 1)] – 29[( – 2)(5) – 3( – 1)] + ( – 4)[( – 11)( – 2) – 3( – 15)]


⇒ D2 = 2[ – 75 – 11] – 29( – 10 + 3) – 4(22 + 45)


⇒ D2 = 2[ – 86] – 29( – 7) – 4(67)


⇒ D2 = – 172 + 203 – 268


⇒ D2 = – 237


And, Solve D3 formed by replacing 1st column by B matrices


Here




Solving determinant, expanding along 1st Row


⇒ D3 = 2[(5)( – 11) – ( – 15)( – 1)] – ( – 3)[( – 11)( – 2) – ( – 15)(3)] + 29[( – 2)( – 1) – (3)(5)]


⇒ D3 = 2[ – 55 – 15] + 3(22 + 45) + 29(2 – 15)


⇒ D3 = 2[ – 70] + 3[67] + 29[ – 13]


⇒ D3 = – 140 + 201 – 377


⇒ D3 = – 316


Thus by Cramer’s Rule, we have




⇒ x = 2


again,




⇒ y = – 3


and,




⇒ z = – 4



Question 18.

Solve the following system of the linear equations by Cramer’s rule:

x + y = 1

x + z = – 6

x – y – 2z = 3


Answer:

Given: - Equations are: –


x + y = 1


x + z = – 6


x – y – 2z = 3


Tip: - Theorem – Cramer’s Rule


Let there be a system of n simultaneous linear equations and with n unknown given by







and let Dj be the determinant obtained from D after replacing the jth column by



Then,


provided that D ≠ 0


Now, here we have


x + y = 1


x + z = – 6


x – y – 2z = 3


So by comparing with theorem, lets find D , D1 and D2



Solving determinant, expanding along 1st Row


⇒ D = 1[(0)( – 2) – (1)( – 1)] – 1[( – 2)(1) – 1] + 0[ – 1 – 0]


⇒ D = 1[0 + 1] – 1[ – 3] – 0[ – 2]


⇒ D = 1 + 3 + 0


⇒ D = 4


Again, Solve D1 formed by replacing 1st column by B matrices


Here




Solving determinant, expanding along 1st Row


⇒ D1 = 1[(0)( – 2) – (1)( – 1)] – 1[( – 2)( – 6) – 3] + 0[6 – 0]


⇒ D1 = 1[0 + 1] – 1[12 – 3] + 0[6]


⇒ D1 = 1[1] – 9 + 0


⇒ D1 = 1 – 9 + 0


⇒ D1 = – 8


Again, Solve D2 formed by replacing 2nd column by B matrices


Here




Solving determinant, expanding along 1st Row


⇒ D2 = 1[( – 6)( – 2) – (1)(3)] – 1[( – 2)(1) – 1] + 0[3 + 6]


⇒ D2 = 1[12 – 3] – 1( – 2 – 1) + 0(9)


⇒ D2 = 9 + 3


⇒ D2 = 12


And, Solve D3 formed by replacing 3rd column by B matrices


Here




Solving determinant, expanding along 1st Row


⇒ D3 = 1[(0)(3) – ( – 1)( – 6)] – 1[(3)(1) – 1( – 6)] + 1[ – 1 + 0]


⇒ D3 = 1[0 – 6] – 1(3 + 6) + 1( – 1)


⇒ D3 = – 6 – 9 – 1


⇒ D3 = – 16


Thus by Cramer’s Rule, we have




⇒ x = – 2


again,




⇒ y = 3


and,




⇒ z = – 4



Question 19.

Solve the following system of the linear equations by Cramer’s rule:

x + y + z + 1 = 0

ax + by + cz + d = 0

a2x + b2y + c2z + d2 = 0


Answer:

Given: - Equations are: –


x + y + z + 1 = 0


ax + by + cz + d = 0


a2x + b2y + c2z + d2 = 0


Tip: - Theorem – Cramer’s Rule


Let there be a system of n simultaneous linear equations and with n unknown given by







and let Dj be the determinant obtained from D after replacing the jth column by



Then,


provided that D ≠ 0


Now, here we have


x + y + z + 1 = 0


ax + by + cz + d = 0


a2x + b2y + c2z + d2 = 0


So by comparing with theorem, lets find D , D1 , D2 and D3



applying,



Take (b – a) from c2 , and (c – a) from c3 common, we get



Solving determinant, expanding along 1st Row


⇒ D = (b – a)(c – a)1[c + a – (b + a)]


⇒ D = (b – a)(c – a)(c + a – b – a)


⇒ D = (b – a)(c – a)(c – b)


⇒ D = (a – b)(b – c)(c – a)


Again, Solve D1 formed by replacing 1st column by B matrices


Here




applying,



Take (b – d) from c2 , and (c – d) from c3 common, we get



Solving determinant, expanding along 1st Row


⇒ D1 = – (b – d)(c – d)1[c + d – (b + d)]


⇒ D1 = – (b – d)(c – d)(c + d – b – d)


⇒ D1 = – (b – d)(c – d)(c – b)


⇒ D1 = – (d – b)(b – c)(c – d)


Again, Solve D2 formed by replacing 2nd column by B matrices


Here




applying,



Take (d – a) from c2 , and (c – a) from c3 common, we get



Solving determinant, expanding along 1st Row


⇒ D2 = – (d – a)(c – a)1[c + a – (d + a)]


⇒ D2 = – (d – a)(c – a)(c + a – d – a)


⇒ D2 = – (d – a)(c – a)(c – d)


⇒ D2 = – (a – d)(d – c)(c – a)


And, Solve D3 formed by replacing 3rd column by B matrices


Here




applying,



Take (b – a) from c2 , and (d – a) from c3 common, we get



Solving determinant, expanding along 1st Row


⇒ D3 = – (b – d)(c – d)1[a + d – (b + a)]


⇒ D3 = – (b – d)(c – d)(a + d – b – a)


⇒ D3 = – (b – d)(c – d)(d – b)


⇒ D3 = – (d – b)(b – d)(c – d)


Thus by Cramer’s Rule, we have




again,




and,





Question 20.

Solve the following system of the linear equations by Cramer’s rule:

x + y + z + w = 2

x – 2y + 2z + 2w = – 6

2x + y – 2z + 2w = – 5

3x – y + 3z – 3w = – 3


Answer:

Given: - Equations are: –


x + y + z + w = 2


x – 2y + 2z + 2w = – 6


2x + y – 2z + 2w = – 5


3x – y + 3z – 3w = – 3


Tip: - Theorem – Cramer’s Rule


Let there be a system of n simultaneous linear equations and with n unknown given by







and let Dj be the determinant obtained from D after replacing the jth column by



Then,


provided that D ≠ 0


Now, here we have


x + y + z + w = 2


x – 2y + 2z + 2w = – 6


2x + y – 2z + 2w = – 5


3x – y + 3z – 3w = – 3


So by comparing with theorem, lets find D, D1, D2,D3 and D4



applying,



Solving determinant, expanding along 1st Row



applying,



⇒ D = 1[ – 6 – 88]


⇒ D = – 94


Again, Solve D1 formed by replacing 1st column by B matrices


Here




applying,



Solving determinant, expanding along 1st Row



⇒ D1 = – 1{( – 10)[6( – 1) – 2( – 4)] – ( – 4)[( – 9)6 – ( – 4)3] + 0}


⇒ D1 = – 1{ – 10[ – 6 + 8] + 4[ – 54 + 12]}


⇒ D1 = – 1{ – 10[2] + 4[ – 42] }


⇒ D1 = 188


Again, Solve D2 formed by replacing 2nd column by B matrices


Here




applying,



Solving determinant, expanding along 1st Row



⇒ D2 = – 1{( – 1)[6( – 9) – 3( – 4)] – ( – 10)[0 – 6( – 4)] + 0[0 + 54]}


⇒ D2 = – 1{ – 1[ – 54 + 12] + 10(24) + 0}


⇒ D2 = – 282


Again, Solve D3 formed by replacing 3rd column by B matrices


Here




applying,



Solving determinant, expanding along 1st Row



⇒ D3 = – 1{( – 1)[ – 3 – ( – 9)2] – ( – 4)[0 – 6( – 9)] + ( – 10)[0 + 6]}


⇒ D3 = – 1{ – 1[15] + 4(54) – 10(6)}


⇒ D3 = – 1{ – 15 + 216 – 60}


⇒ D3 = – 141


And, Solve D4 formed by replacing 4th column by B matrices


Here




applying,



Solving determinant, expanding along 1st Row



⇒ D4 = ( – 3)[( – 9)( – 4) – 0] – 1[9 – ( – 4)( – 9)] + ( – 8)[0 – 16]


⇒ D4 = – 3[36] – 1(9 – 36) – 8( – 16)


⇒ D4 = – 108 + 27 + 128


⇒ D4 = 47


Thus by Cramer’s Rule, we have




⇒ x = – 2


again,




⇒ y = 3


again,





And,






Question 21.

Solve the following system of the linear equations by Cramer’s rule:

2x – 3z + w = 1

x – y + 2w = 1

– 3y + z + w = 1

x + y + z = 1


Answer:

Given: - Equations are: –


2x – 3z + w = 1


x – y + 2w = 1


– 3y + z + w = 1


x + y + z = 1


Tip: - Theorem – Cramer’s Rule


Let there be a system of n simultaneous linear equations and with n unknown given by







and let Dj be the determinant obtained from D after replacing the jth column by



Then,


provided that D ≠ 0


Now, here we have


2x – 3z + w = 1


x – y + 2w = 1


– 3y + z + w = 1


x + y + z = 1


So by comparing with theorem, lets find D, D1, D2,D3 and D4



applying,



Solving determinant, expanding along 4th Row



applying,



expanding along 3rd row


⇒ D = – 1[ – 3 – ( – 6)4]


⇒ D = – 21


Again, Solve D1 formed by replacing 1st column by B matrices


Here




applying,



Solving determinant, expanding along 1st Row



⇒ D1 = ( – 1)[(4)( – 1) – 0(4)] – (3)[( – 3)( – 1) – 0] + 1[ – 12 – 4]


⇒ D1 = – 1[ – 4 – 0] – 3[3 – 0] – 16


⇒ D1 = 4 – 9 – 16


⇒ D1 = – 21


Again, Solve D2 formed by replacing 2nd column by B matrices


Here




applying,



Solving determinant, expanding along 4th Row



⇒ D2 = – 1{( – 1)[1( – 1) – 1(2)] – ( – 5)[0 – 1(2)] + 1[0 – ( – 1)]}


⇒ D2 = – 1{ – 1[ – 1 – 2] + 5( – 2) + 1}


⇒ D2 = 6


Again, Solve D3 formed by replacing 3rd column by B matrices


Here




applying,



Solving determinant, expanding along 4th Row



⇒ D3 = – 1{( – 2)[0 – (1)2] – ( – 1)[ – 2 – ( – 3)(2)] + 1[ – 2 – 0]}


⇒ D3 = – 1{ – 2[ – 2] + 1( – 2 + 6) + 1( – 2)}


⇒ D3 = – 1{4 + 4 – 2}


⇒ D3 = – 6


And, Solve D4 formed by replacing 4th column by B matrices


Here




applying,



Solving determinant, expanding along 4th Row



⇒ D4 = ( – 1){( – 2)[( – 1)1 – 0] – ( – 5)[ – 2 – 0] + ( – 1)[ – 2 – 3]}


⇒ D4 = ( – 1){2 – 10 + 5}


⇒ D4 = 3


⇒ D4 = 3


Thus by Cramer’s Rule, we have




⇒ x = 1


again,





again,





And,






Question 22.

Show that each of the following systems of linear equations is inconsistent:

2x – y = 5

4x – 2y = 7


Answer:

Given: - Two equation 2x – y = 5 and 4x – 2y = 7


Tip: - We know that


For a system of 2 simultaneous linear equation with 2 unknowns


(i) If D ≠ 0, then the given system of equations is consistent and has a unique solution given by



(ii) If D = 0 and D1 = D2 = 0, then the system is consistent and has infinitely many solution.


(iii) If D = 0 and one of D1 and D2 is non – zero, then the system is inconsistent.


Now,


We have,


2x – y = 5


4x – 2y = 7


Lets find D



⇒ D = – 4 + 4


⇒ D = 0


Again, D1 by replacing 1st column by B


Here




⇒ D1 = – 10 + 7


⇒ D1 = – 3


And, D2 by replacing 2nd column by B


Here




⇒ D2 = 14 – 20


⇒ D2 = – 6


So, here we can see that


D = 0 and D1 and D2 are non – zero


Hence the given system of equation is inconsistent.



Question 23.

Show that each of the following systems of linear equations is inconsistent:

3x + y = 5

– 6x – 2y = 9


Answer:

Given: - Two equation 3x + y = 5 and – 6x – 2y = 9


Tip: - We know that


For a system of 2 simultaneous linear equation with 2 unknowns


(i) If D ≠ 0, then the given system of equations is consistent and has a unique solution given by



(ii) If D = 0 and D1 = D2 = 0, then the system is consistent and has infinitely many solution.


(iii) If D = 0 and one of D1 and D2 is non – zero, then the system is inconsistent.


Now,


We have,


3x + y = 5


– 6x – 2y = 9


Lets find D



⇒ D = – 6 – 6


⇒ D = 0


Again, D1 by replacing 1st column by B


Here




⇒ D1 = – 10 – 9


⇒ D1 = – 19


And, D2 by replacing 2nd column by B


Here




⇒ D2 = 27 + 30


⇒ D2 = 57


So, here we can see that


D = 0 and D1 and D2 are non – zero


Hence the given system of equation is inconsistent.



Question 24.

Show that each of the following systems of linear equations is inconsistent:

3x – y + 2z = 3

2x + y + 3z = 5

x – 2y – z = 1


Answer:

Given: - Three equation


3x – y + 2z = 3


2x + y + 3z = 5


x – 2y – z = 1


Tip: - We know that


For a system of 3 simultaneous linear equation with 3 unknowns


(i) If D ≠ 0, then the given system of equations is consistent and has a unique solution given by



(ii) If D = 0 and D1 = D2 = D3 = 0, then the given system of equation may or may not be consistent. However if consistent, then it has infinitely many solutions.


(iii) If D = 0 and at least one of the determinants D1, D2 and D3 is non – zero, then the system is inconsistent.


Now,


We have,


3x – y + 2z = 3


2x + y + 3z = 5


x – 2y – z = 1


Lets find D



Expanding along 1st row


⇒ D = 3[ – 1 – 3( – 2)] – ( – 1)[( – 1)2 – 3] + 2[ – 4 – 1]


⇒ D = 3[5] + 1[ – 5] + 2[ – 5]


⇒ D = 0


Again, D1 by replacing 1st column by B


Here




⇒ D1 = 3[ – 1 – 3( – 2)] – ( – 1)[( – 1)5 – 3] + 2[ – 10 – 1]


⇒ D1 = 3[5] + [ – 8] + 2[ – 11]


⇒ D1 = 15 – 8 – 22


⇒ D1 = – 15


⇒ D1 ≠ 0


So, here we can see that


D = 0 and D1 is non – zero


Hence the given system of equation is inconsistent.


Hence Proved



Question 25.

Show that each of the following systems of linear equations is inconsistent:

x + y + z = 3

2x – y + z = 2

3x + 6y + 5z = 20.


Answer:

Given: - Three equation


x + y + z = 3


2x – y + z = 2


3x + 6y + 5z = 20.


Tip: - We know that


For a system of 3 simultaneous linear equation with 3 unknowns


(i) If D ≠ 0, then the given system of equations is consistent and has a unique solution given by



(ii) If D = 0 and D1 = D2 = D3 = 0, then the given system of equation may or may not be consistent. However if consistent, then it has infinitely many solution.


(iii) If D = 0 and at least one of the determinants D1, D2 and D3 is non – zero, then the system is inconsistent.


Now,


We have,


x + y + z = 3


2x – y + z = 2


3x + 6y + 5z = 20.


Lets find D



Expanding along 1st row


⇒ D = 1[ – 5 – 1(6)] – (1)[(5)2 – 3] + 1[12 + 3]


⇒ D = 1[ – 11] – 1[7] + 1[15]


⇒ D = – 3


So, here we can see that


D ≠ 0


Hence the given system of equation is consistent.



Question 26.

Show that each of the following systems of linear equations has infinite number of solutions and solve:

x – y + z = 3

2x + y – z = 2

– x – 2y + 2z = 1


Answer:

Given: - Three equation


x – y + z = 3


2x + y – z = 2


– x – 2y + 2z = 1


Tip: - We know that


For a system of 3 simultaneous linear equation with 3 unknowns


(i) If D ≠ 0, then the given system of equations is consistent and has a unique solution given by



(ii) If D = 0 and D1 = D2 = D3 = 0, then the given system of equation may or may not be consistent. However if consistent, then it has infinitely many solution.


(iii) If D = 0 and at least one of the determinants D1, D2 and D3 is non – zero, then the system is inconsistent.


Now,


We have,


x – y + z = 3


2x + y – z = 2


– x – 2y + 2z = 1


Lets find D



Expanding along 1st row


⇒ D = 1[2 – ( – 1)( – 2)] – ( – 1)[(2)2 – (1)] + 1[ – 4 – ( – 1)]


⇒ D = 1[0] + 1[3] + [ – 3]


⇒ D = 0


Again, D1 by replacing 1st column by B


Here




⇒ D1 = 3[2 – ( – 1)( – 2)] – ( – 1)[(2)2 – ( – 1)] + 1[ – 4 – 1]


⇒ D1 = 3[2 – 2] + [4 + 1] + 1[ – 5]


⇒ D1 = 0 + 5 – 5


⇒ D1 = 0


Also, D2 by replacing 2nd column by B


Here




⇒ D2 = 1[4 – ( – 1)(1)] – (3)[(2)2 – (1)] + 1[2 – ( – 2)]


⇒ D2 = 1[4 + 1] – 3[4 – 1] + 1[4]


⇒ D2 = 5 – 9 + 4


⇒ D2 = 0


Again, D3 by replacing 3rd column by B


Here




⇒ D3 = 1[1 – ( – 2)(2)] – ( – 1)[(2)1 – 2( – 1)] + 3[2( – 2) – 1( – 1)]


⇒ D3 = [1 + 4] + [2 + 2] + 3[ – 4 + 1]


⇒ D3 = 5 + 4 – 9


⇒ D3 = 0


So, here we can see that


D = D1 = D2 = D3 = 0


Thus,


Either the system is consistent with infinitely many solutions or it is inconsistent.


Now, by 1st two equations, written as


x – y = 3 – z


2x + y = 2 + z


Now by applying Cramer’s rule to solve them,


New D and D1, D2



⇒ D = 1 + 2


⇒ D = 3


Again, D1 by replacing 1st column with




⇒ D1 = 3 – z – ( – 1)(2 + z)


⇒ D1 = 5


Again, D2 by replacing 2nd column with




⇒ D2 = 2 + z – 2 (3 – z)


⇒ D2 = – 4 + 3z


Hence, using Cramer’s rule




again,




Let, z = k


Then


And z = k


By changing value of k you may get infinite solutions



Question 27.

Show that each of the following systems of linear equations has infinite number of solutions and solve:

x + 2y = 5

3x + 6y = 15


Answer:

Given: - Two equation x + 2y = 5 and 3x + 6y = 15


Tip: - We know that


For a system of 2 simultaneous linear equation with 2 unknowns


(iv) If D ≠ 0, then the given system of equations is consistent and has a unique solution given by



(v) If D = 0 and D1 = D2 = 0, then the system is consistent and has infinitely many solution.


(vi) If D = 0 and one of D1 and D2 is non – zero, then the system is inconsistent.


Now,


We have,


x + 2y = 5


3x + 6y = 15


Lets find D



⇒ D = – 6 – 6


⇒ D = 0


Again, D1 by replacing 1st column by B


Here




⇒ D1 = 30 – 30


⇒ D1 = 0


And, D2 by replacing 2nd column by B


Here




⇒ D2 = 15 – 15


⇒ D2 = 0


So, here we can see that


D = D1 = D2 = 0


Thus,


The system is consistent with infinitely many solutions.


Let


y = k


then,


⇒ x + 2y = 5


⇒ x = 5 – 2k


By changing value of k you may get infinite solutions



Question 28.

Show that each of the following systems of linear equations has infinite number of solutions and solve:

x + y – z = 0

x – 2y + z = 0

3x + 6y – 5z = 0


Answer:

Given: - Three equation


x + y – z = 0


x – 2y + z = 0


3x + 6y – 5z = 0


Tip: - We know that


For a system of 3 simultaneous linear equation with 3 unknowns


(i) If D ≠ 0, then the given system of equations is consistent and has a unique solution given by



(ii) If D = 0 and D1 = D2 = D3 = 0, then the given system of equation may or may not be consistent. However if consistent, then it has infinitely many solution.


(iii) If D = 0 and at least one of the determinants D1, D2 and D3 is non – zero, then the system is inconsistent.


Now,


We have,


x + y – z = 0


x – 2y + z = 0


3x + 6y – 5z = 0


Lets find D



Expanding along 1st row


⇒ D = 1[10 – (6)1] – (1)[( – 5)1 – (1)3] + ( – 1)[6 – ( – 2)3]


⇒ D = 1[4] – 1[ – 8] – [12]


⇒ D = 0


Again, D1 by replacing 1st column by B


Here




As one column is zero its determinant is zero


⇒ D1 = 0


Also, D2 by replacing 2nd column by B


Here




As one column is zero its determinant is zero


⇒ D2 = 0


Again, D3 by replacing 3rd column by B


Here




As one column is zero its determinant is zero


⇒ D3 = 0


So, here we can see that


D = D1 = D2 = D3 = 0


Thus,


Either the system is consistent with infinitely many solutions or it is inconsistent.


Now, by 1st two equations, written as


x + y = z


x – 2y = – z


Now by applying Cramer’s rule to solve them,


New D and D1, D2



⇒ D = – 2 – 1


⇒ D = – 3


Again, D1 by replacing 1st column with




⇒ D1 = – 2z – 1( – z)


⇒ D1 = – z


Again, D2 by replacing 2nd column with




⇒ D2 = – z – z


⇒ D2 = – 2z


Hence, using Cramer’s rule




Let, z = k


Then


again,





And z = k


By changing value of k you may get infinite solutions



Question 29.

Show that each of the following systems of linear equations has infinite number of solutions and solve:

2x + y – 2z = 4

x – 2y + z = – 2

5x – 5y + z = – 2


Answer:

Given: - Three equation


2x + y – 2z = 4


x – 2y + z = – 2


5x – 5y + z = – 2


Tip: - We know that


For a system of 3 simultaneous linear equation with 3 unknowns


(i) If D ≠ 0, then the given system of equations is consistent and has a unique solution given by



(ii) If D = 0 and D1 = D2 = D3 = 0, then the given system of equation may or may not be consistent. However if consistent, then it has infinitely many solution.


(iii) If D = 0 and at least one of the determinants D1, D2 and D3 is non – zero, then the system is inconsistent.


Now,


We have,


2x + y – 2z = 4


x – 2y + z = – 2


5x – 5y + z = – 2


Lets find D



Expanding along 1st row


⇒ D = 2[ – 2 – ( – 5)(1)] – (1)[(1)1 – 5(1)] + ( – 2)[ – 5 – 5( – 2)]


⇒ D = 2[3] – 1[ – 4] – 2[5]


⇒ D = 0


Again, D1 by replacing 1st column by B


Here




⇒ D1 = 4[ – 2 – ( – 5)(1)] – (1)[( – 2)1 – ( – 2)(1)] + ( – 2)[( – 2)( – 5) – ( – 2)( – 2)]


⇒ D1 = 4[ – 2 + 5] – [ – 2 + 2] – 2[6]


⇒ D1 = 12 + 0 – 12


⇒ D1 = 0


Also, D2 by replacing 2nd column by B


Here




⇒ D2 = 2[ – 2 – ( – 2)(1)] – (4)[(1)1 – (5)] + ( – 2)[ – 2 – 5( – 2)]


⇒ D2 = 2[ – 2 + 2] – 4[ – 4] + ( – 2)[8]


⇒ D2 = 0 + 16 – 16


⇒ D2 = 0


Again, D3 by replacing 3rd column by B


Here




⇒ D3 = 2[4 – ( – 2)( – 5)] – (1)[( – 2)1 – 5( – 2)] + 4[1( – 5) – 5( – 2)]


⇒ D3 = 2[ – 6] – [8] + 4[ – 5 + 10]


⇒ D3 = – 12 – 8 + 20


⇒ D3 = 0


So, here we can see that


D = D1 = D2 = D3 = 0


Thus,


Either the system is consistent with infinitely many solutions or it is inconsistent.


Now, by 1st two equations, written as


x – 2y = – 2 – z


5x – 5y = – 2 – z


Now by applying Cramer’s rule to solve them,


New D and D1, D2



⇒ D = – 5 + 10


⇒ D = 5


Again, D1 by replacing 1st column with




⇒ D1 = 10 + 5z – ( – 2)( – 2 – z)


⇒ D1 = 6 + 3z


Again, D2 by replacing 2nd column with




⇒ D2 = – 2 – z – 5 ( – 2 – z)


⇒ D2 = 8 + 4z


Hence, using Cramer’s rule




again,




Let, z = k


Then




And z = k


By changing value of k you may get infinite solutions



Question 30.

Show that each of the following systems of linear equations has infinite number of solutions and solve:

x – y + 3z = 6

x + 3y – 3z = – 4

5x + 3y + 3z = 10


Answer:

Given: - Three equation


x – y + 3z = 6


x + 3y – 3z = – 4


5x + 3y + 3z = 10


Tip: - We know that


For a system of 3 simultaneous linear equation with 3 unknowns


(iv) If D ≠ 0, then the given system of equations is consistent and has a unique solution given by



(v) If D = 0 and D1 = D2 = D3 = 0, then the given system of equation may or may not be consistent. However if consistent, then it has infinitely many solution.


(vi) If D = 0 and at least one of the determinants D1, D2 and D3 is non – zero, then the system is inconsistent.


Now,


We have,


x – y + 3z = 6


x + 3y – 3z = – 4


5x + 3y + 3z = 10


Lets find D



Expanding along 1st row


⇒ D = 1[9 – ( – 3)(3)] – ( – 1)[(3)1 – 5( – 3)] + 3[3 – 5(3)]


⇒ D = 1[18] + 1[18] + 3[12]


⇒ D = 0


Again, D1 by replacing 1st column by B


Here




⇒ D1 = 6[9 – ( – 3)(3)] – ( – 1)[( – 4)3 – 10( – 3)] + 3[ – 12 – 30]


⇒ D1 = 6[9 + 9] + [ – 12 + 30] + 3[ – 42]


⇒ D1 = 6[18] + 18 – 3[42]


⇒ D1 = 0


Also, D2 by replacing 2nd column by B


Here




⇒ D2 = 1[ – 12 – ( – 3)10] – 6[3 – 5( – 3)] + 3[10 – 5( – 4)]


⇒ D2 = [ – 12 + 30] – 6[3 + 15] + 3[10 + 20]


⇒ D2 = 18 – 6[18] + 3[30]


⇒ D2 = 0


Again, D3 by replacing 3rd column by B


Here




⇒ D3 = 1[30 – ( – 4)(3)] – ( – 1)[(10 – 5( – 4)] + 6[3 – 15]


⇒ D3 = 1[30 + 12] + 1[10 + 20] + 6[ – 12]


⇒ D3 = 42 + 30 – 72


⇒ D3 = 0


So, here we can see that


D = D1 = D2 = D3 = 0


Thus,


Either the system is consistent with infinitely many solutions or it is inconsistent.


Now, by 1st two equations, written as


x – y = 6 – 3z


x + 3y = – 4 + 3z


Now by applying Cramer’s rule to solve them,


New D and D1, D2



⇒ D = 3 + 1


⇒ D = 4


Again, D1 by replacing 1st column with




⇒ D1 = 18 – 9z – ( – 1)( – 4 + 3z)


⇒ D1 = 14 – 5z


Again, D2 by replacing 2nd column with




⇒ D2 = – 4 + 3z – (6 – 3z)


⇒ D2 = – 10 + 6z


Hence, using Cramer’s rule





again,





Let, z = k


Then




And z = k


By changing value of k you may get infinite solutions



Question 31.

A salesman has the following record of sales during three months for three items A,B and C which have different rates of commission.



Find out the rates of commission on items A,B and C by using determinant method.


Answer:

Given: - Record of sales during three months


Let, rates of commissions on items A,B and C be x, y and z respectively.


Now, we can arrange this model in linear equation system


Thus, we have


90x + 100y + 20z = 800


130x + 50y + 40z = 900


60x + 100y + 30z = 850


Here



Applying,



Solving determinant, expanding along 2nd column


⇒ D = 50[( – 50)( – 170) – ( – 200)( – 60)]


⇒ D = 50[8500 – 12000]


⇒ D = – 175000


Again, Solve D1 formed by replacing 1st column by B matrices


Here




Applying,



Solving determinant, expanding along 2nd column


⇒ D1 = 50[( – 1000)( – 500) – ( – 950)( – 60)]


⇒ D1 = 50[50000 – 57000]


⇒ D1 = – 350000


Again, Solve D2 formed by replacing 2nd column by B matrices


Here




Applying,



Solving determinant, expanding along 1st Row


⇒ D2 = 20[17500 – 52500]


⇒ D2 = – 700000


And, Solve D3 formed by replacing 3rd column by B matrices


Here




Applying,



Solving determinant, expanding along 1st Row


⇒ D3 = 50[161500 – 200000]


⇒ D3 = – 1925000


Thus by Cramer’s Rule, we have




⇒ x = 2


again,




⇒ y = 4


and,




z = 11


Thus rates of commission of items A, B and C are 2%, 4% and 11% respectively.



Question 32.

An automobile company uses three types of steel S1, S2 and S3 for producing three types of cars C1, C2 and C3. Steel requirements (in tons) for each type of cars are given below:



Using Cramer’s rule, find the number of cars of each type which can be produced using 29, 13 and 16 tonnes of steel of three types respectively.


Answer:

Given: - Steel requirement for each car is given


Let, Number of cars produced by steel type C1, C2 and C3 be x, y and z respectively.


Now, we can arrange this model in linear equation system


Thus, we have


2x + 3y + 4z = 29


x + y + 2z = 13


3x + 2y + z = 16


Here



Applying,



Solving determinant, expanding along 3rd column


⇒ D = 1[30 – 25]


⇒ D = 5


⇒ D = 5


Again, Solve D1 formed by replacing 1st column by B matrices


Here




Applying,



Solving determinant, expanding along 3rd column


⇒ D1 = 1[( – 35)( – 3) – ( – 5)( – 19)]


⇒ D1 = 1[105 – 95]


⇒ D1 = 10


Again, Solve D2 formed by replacing 2nd column by B matrices


Here




Applying,



Solving determinant, expanding along 3rd column


⇒ D2 = 1[190 – 175]


⇒ D2 = 15


And, Solve D3 formed by replacing 3rd column by B matrices


Here




Applying,



Solving determinant, expanding along 1st column


⇒ D3 = – 1[ – 23 – ( – 1)3]


⇒ D3 = 20


Thus by Cramer’s Rule, we have




⇒ x = 2


again,




⇒ y = 3


and,




⇒ z = 4


Thus Number of cars produced by type C1, C2 and C3 are 2, 3 and 4 respectively.




Exercise 6.5
Question 1.

Solve each of the following systems of homogeneous linear equations:

x + y – 2z = 0

2x + y – 3z = 0

5x + 4y – 9z = 0


Answer:

Given Equations:


x + y – 2z = 0


2x + y – 3z = 0


5x + 4y – 9z = 0


Any system of equation can be written in matrix form as AX = B


Now finding the Determinant of these set of equations,




= 1(1×(– 9) – 4×(– 3)) – 1(2×(– 9) – 5×(– 3)) – 2(4×2 – 5×1)


= 1(– 9 + 12) – 1(– 18 + 15) – 2(8 – 5)


= 1×3 –1 × (– 3) – 2×3


= 3 + 3 – 6


= 0


Since D = 0, so the system of equation has infinite solution.


Now let z = k


⇒ x + y = 2k


And 2x + y = 3k


Now using the cramer’s rule






x = k


similarly,






y = k


Hence, x = y = z = k.



Question 2.

Solve each of the following systems of homogeneous linear equations:

2x + 3y + 4z = 0

X + y + z = 0

2x + 5y – 2z = 0


Answer:

Given Equations:


2x + 3y + 4z = 0


X + y + z = 0


2x + 5y – 2z = 0


Any system of equation can be written in matrix form as AX = B


Now finding the Determinant of these set of equations,




= 2(1×(– 2) – 1×5) – 3(1×(– 2) – 2×1) + 4(1×5 – 2×1)


= 2(– 2 – 5) – 3(– 2 – 2) + 4(5 – 2)


= 1×(– 7) – 3 × (– 4) + 4×3


= – 7 + 12 + 12


= 17


Since D ≠ 0, so the system of equation has infinite solution.


Therefore the system of equation has only solution as x = y = z = 0.



Question 3.

Solve each of the following systems of homogeneous linear equations:

3x + y + z = 0

x – 4y3z = 0

2x + 5y – 2z = 0


Answer:

Given Equations:


3x + y + z = 0


x – 4y + 3z = 0


2x + 5y – 2z = 0


Any system of equation can be written in matrix form as AX = B


Now finding the Determinant of these set of equations,




= 3(– 4×(– 2) – 3×5) – 1(1×(– 2) – 3×2) + 1(1×5 – 2×(– 4))


= 3(8 – 15) – 1(– 2 – 6) + 1(5 + 8)


= 3×(– 7) –1 × (– 8) + 1×13


= – 21 + 8 + 13


= 0


Since D = 0, so the system of equation has infinite solution.


Now let z = k


⇒ 3x + y = – k


And x – 4y = – 3k


Now using the cramer’s rule






similarly,








Question 4.

Find the real values of λ for which the followings system of linear equations has non – trivial solutions. Also, find the non – trivial solutions

2λx – 2y + 3z = 0

X + λy + 2z = 0

2x + λz = 0


Answer:

Given Equations:


2λx – 2y + 3z = 0


x + λy + 2z = 0


2x + λz = 0


For trivial solution D = 0




= 2λ (λ×λ – 0×2) + 2(1×λ – 2×2) + 3(1×0 – 2×λ)


= 2λ (λ2 – 0) + 2(λ – 4) + 3(0 – 2λ)


= 2λ3 + 2λ – 8 – 6λ


= 2λ3 + 4λ – 8


Now D = 0


3 – 4λ – 8 = 0


3 – 4λ = 8


λ(λ2 – 2) = 4


Hence λ = 2


Now let z = k


⇒ 4x – 2y = – 3k


And x + 2y = – 2k


Now using the cramer’s rule






x = – k


similarly,









Question 5.

If a,b,c are non – zero real numbers and if the system of equations

(a – 1) x = y + z

(b – 1) y = z + x

(c – 1) z = x + y

Has a non – trivial solution, then prove that ab + bc + ca = abc.


Answer:

Given Equations:


(a – 1) x = y + z


(b – 1) y = z + x


(c – 1) z = x + y


Rearranging these equations


(a – 1)x – y – z = 0


– x + (b – 1)y – z = 0


– x – y + (c – 1)z = 0


For trivial solution D = 0




= (a – 1)((b – 1)(c – 1) – (– 1)×(– 1)) + 1(– 1(c – 1) – (– 1)×(– 1)) – 1((– 1)×(– 1) + (b – 1))


= (a – 1)(bc –b –c + 1 – 1) + (1 – c – 1) – 1(1 + b – 1))


= (a – 1)(bc –b –c) –c –b


= abc –ab –ac –bc + b + c – b – c


= abc –ab –ac –bc


Now D = 0


⇒ abc – ab – ac – bc = 0


⇒ abc = ab + bc + ac


Hence proved.




Mcq
Question 1.

Mark the correct alternative in the following:

If A and B are square matrices or order 2, then det (A + B) = 0 is possible only when

A. det (A) = 0 or det (B) = 0

B. det (A) + det (B) = 0

C. det (A) = 0 and det (B) = 0

D. A + B = 0


Answer:

We are given that,

Matrices A and B are square matrices.


Order of matrix A = 2


Order of matrix B = 2


Det (A + B) = 0


We need to find the condition at which det (A + B) = 0.


Let,


Matrix A = [aij]


Matrix B = [bij]


Since their orders are same, we can express matrices A and B as


A + B = [aij + bij]


⇒ |A + B| = |aij + bij| …(i)


Also, we know that


Det (A + B) = 0


That is, |A + B| = 0


From (i),


|aij + bij| = 0


If


⇒ [aij + bij] = 0


Each corresponding element is 0.


⇒ A + B = 0


Thus, det (A + B) = 0 is possible when A + B = 0 .


Question 2.

Mark the correct alternative in the following:

Which of the following is not correct?

A. |A| = |AT|, where A = [aij]3 × 3

B. |kA| = k3 |A|, where A = [aij]3 × 3

C. If A is a skew-symmetric matrix of odd order, then |A| = 0

D.


Answer:

We are given that,


A = [aij]3×3


That is, order of matrix A = 3


Example:


Let,



Take determinant of A.


Determinant of 3 × 3 matrices is found as,




So,








⇒ |A| = -11


The transpose of a matrix is a new matrix whose rows are the columns of the original.


So,



Determinant of AT:








So, we can conclude that,


|A| = |AT|, where A = [aij]3×3.


Option (B) is correct.


|kA| = k3|A|, where A = [aij]3×3


Example:


Let k = 2.


And,



Take Left Hand Side of the equation:


LHS = |kA|



Multiply 2 by each term of the matrix.





⇒ LHS = 4(4 × 2 – 6 × 4) – 6(2 × 2 – 6 × 6) + 8(2 × 4 – 4 × 6)


⇒ LHS = 4(8 – 24) – 6(4 – 36) + 8(8 – 24)


⇒ LHS = 4(-16) – 6(-32) + 8(-16)


⇒ LHS = -64 + 192 – 128


⇒ LHS = 0


Take Right Hand Side of the equation:


RHS = k3|A|





⇒ RHS = 8 [2(2 × 1 – 3 × 2) – 3(1 × 1 – 3 × 3) + 4(1 × 2 – 2 × 3)]


⇒ RHS = 8 [2(2 – 6) – 3(1 – 9) + 4(2 – 6)]


⇒ RHS = 8 [2(-4) – 3(-8) + 4(-4)]


⇒ RHS = 8 [-8 + 24 – 16]


⇒ RHS = 8 × 0


⇒ RHS = 0


Since, LHS = RHS.


We can conclude that,


|kA| = k3|A|, where A = [aij]3×3


Option (C) is also correct.


If A is a skew-symmetric matrix of odd order, then |A| = 0.


If the transpose of a matrix is equal to the negative of itself, the matrix is said to be skew symmetric. In other words, AT = -A.


Example,


Let a matrix of odd order 3 × 3 be,



Take determinant of A.








Thus, we can conclude that


If A is a skew-symmetric matrix of odd order, then |A| = 0.


Option (D) is incorrect.


Let a = 1, b = 3, c = 3, d = -4, e = -2, f = 5, g = 0 and h = 2.


Take Left Hand Side,





⇒ LHS = 4 × 2 – (-1) × 3


⇒ LHS = 8 + 3


⇒ LHS = 11


Take Right Hand Side,




⇒ RHS = (1 × 0 – 3 × (-2)) + (3 × 2 – (-4) × 5)


⇒ RHS = (0 + 6) + (6 + 20)


⇒ RHS = 6 + 26


⇒ RHS = 32


Since, LHS ≠ RHS. Then, we can conclude that,



Question 3.

Mark the correct alternative in the following:

If and Cij is cofactor of aij in A, then value of |A| is given by

A. a11C31 + a12C32 + a13C33

B. a11C11 + a12C21 + a13C31

C. a21C11 + a22C12 + a23C13

D. a11C11 + a21C21 + a31C31


Answer:

Let us understand what cofactor of an element is.

A cofactor is the number you get when you remove the column and row of a designated element in a matrix, which is just a numerical grid in the form of a rectangle or a square. The cofactor is always preceded by a positive (+) or negative (-) sign, depending whether the element is in a + or - position. It is



Let us recall how to find the cofactor of any element:


If we are given with,



Cofactor of any element say a11 is found by eliminating first row and first column.



⇒ Cofactor of a11 = a22 × a33 – a23 × a32


The sign of cofactor of a11 is (+).


And, cofactor of any element, say a12 is found by eliminating first row and second column.



⇒ Cofactor of a12 = a21 × a33 – a23 × a31


The sign of cofactor of a12 is (-).


We are given that,



And Cij is the cofactor of aij in A.


Determinant of 3 × 3 matrix is given as,



Or,



Or using the definition of cofactors,



Thus, proved.


Question 4.

Mark the correct alternative in the following:

Which of the following is not correct in a given determinant of A, where A = [aij]3 × 3.

A. Order of minor is less than order of the det (A)

B. Minor of an element can never be equal to cofactor of the same element

C. Value of a determinant is obtained by multiplying elements of a row or column by corresponding cofactors

D. Order of minors and cofactors of elements of A is same


Answer:

For option (A),

A minor is the determinant of the square matrix formed by deleting one row and one column from some larger square matrix.


So, the order of minor is always less than the order of determinant.


Thus, option (A) is correct.


For option (B),


A cofactor is the number you get when you remove the column and row of a designated element in a matrix, which is just a numerical grid in the form of a rectangle or a square.


A minor is the determinant of the square matrix formed by deleting one row and one column from some larger square matrix.


Since, the definition of cofactor and minor is same, then we can conclude that


Minor of an element is always equal to cofactor of the same element.


Thus, option (B) is incorrect.


For option (C),


Determinant of 3 × 3 matrix is given as,



Or,



Or using the definition of cofactors,



Thus, option (C) is correct.


For option (D),


A cofactor is the number you get when you remove the column and row of a designated element in a matrix, which is just a numerical grid in the form of a rectangle or a square.


A minor is the determinant of the square matrix formed by deleting one row and one column from some larger square matrix.


Since, the definition of cofactor and minor is same, then we can say that,


Minor of an element is always equal to cofactor of the same element.


⇒ The order of the minor and cofactor of A is same. (where A is some matrix)


Thus, option (D) is correct.


Question 5.

Mark the correct alternative in the following:

Let . Then, the value of 5a + 4b + 3c + 2d + e is equal to

A. 0

B. -16

C. 16

D. none of these


Answer:

We are given that,


We need to find the value of 5a + 4b + 3c + 2d + e.


Determinant of 3 × 3 matrix is given as,




So,







Since,



⇒ x4 – x3 – 12x2 + 12x = ax4 + bx3 + cx2 + dx + e


Comparing the left hand side and right hand side of the equation, we get


a = 1


b = -1


c = -12


d = 12


e = 0


Putting these values in 5a + 4b + 3c + 2d + e, we get


5a + 4b + 3c + 2d + e = 5(1) + 4(-1) + 3(-12) + 2(12) + 0


⇒ 5a + 4b + 3c + 2d + e = 5 – 4 – 36 + 24


⇒ 5a + 4b + 3c + 2d + e = 25 – 36


⇒ 5a + 4b + 3c + 2d + e = -11


Thus, the values of 5a + 4b + 3c + 2d + e is -11.


Question 6.

Mark the correct alternative in the following:

The value of the determinant is independent of

A. n

B. a

C. x

D. none of these


Answer:

Let us solve the determinant.


We know that,


Determinant of 3 × 3 matrix is given as,




So,




By trigonometric identity, we have


sin (α – β) = sin α cos β – cos α sin β


So, we can write





Note that, the result has ‘a’ as well as ‘x’, but doesn’t contain ‘n’.


Thus, the determinant is independent of n.


Question 7.

Mark the correct alternative in the following:

If , then

A. Δ1 + Δ2 = 0

B. Δ1 + 2Δ2 = 0

C. Δ1 = Δ2

D. none of these


Answer:

We are given that,


Let us find the determinants ∆1 and ∆2.


We know that,


Determinant of 3 × 3 matrix is given as,




So,




⇒ ∆1 = (b × c2 – c × b2) – (a × c2 – c × a2) + (a × b2 – b × a2)


⇒ ∆1 = bc2 – b2c – ac2 + a2c + ab2 – a2b …(i)


Also,




⇒ ∆2 = (ca × c – b × ab) – bc(1 × c – b × 1) + a(1 × ab – ca × 1)


⇒ ∆2 = ac2 – ab2 – bc(c – b) + a(ab – ac)


⇒ ∆2 = ac2 – ab2 – bc2 + b2c + a2b – a2c …(ii)


Checking Option (A).


Adding ∆1 and ∆2 by using values from (i) and (ii),


1 + ∆2 = (bc2 – b2c – ac2 + a2c + ab2 – a2b) + (ac2 – ab2 – bc2 + b2c + a2b – a2c)


⇒ ∆1 + ∆2 = bc2 – bc2 – b2c + b2c – ac2 + ac2 + ab2 – ab2 – a2b + a2b


⇒ ∆1 + ∆2 = 0


Thus, option (A) is correct.


Checking Option (B).


Multiplying 2 by (ii),


2∆2 = 2(ac2 – ab2 – bc2 + b2c + a2b – a2c)


⇒ 2∆2 = 2ac2 – 2ab2 – 2bc2 + 2b2c + 2a2b – 2a2c …(iii)


Then, adding 2∆2 with ∆1,


1 + 2∆2 = (bc2 – b2c – ac2 + a2c + ab2 – a2b) + (2ac2 – 2ab2 – 2bc2 + 2b2c + 2a2b – 2a2c)


⇒ ∆1 + 2∆2 = bc2 – 2bc2 – b2c + 2b2c – ac2 + 2ac2 + ab2 – 2ab2 – a2b + 2a2b


⇒ ∆1 + 2∆2 = -bc2 + b2c + ac2 – ab2 + a2b


⇒ ∆1 + 2∆2 ≠ 0


Thus, option (B) is not correct.


Checking option (C).


Obviously, ∆1 ≠ ∆2


Since, by (i) and (ii), we can notice ∆1 and ∆2 have different values.


Thus, option (C) is not correct.


Question 8.

Mark the correct alternative in the following:

If and , then n equals

A. 4

B. 6

C. 8

D. none of these


Answer:

We are given that,



We need to find the value of e.


We know that,


Determinant of 3 × 3 matrix is given as,



Question 9.

Mark the correct alternative in the following:

Let be an identify in x, were a, b, c, d, e are independent of x. Then the value of e is

A. 4

B. 0

C. 1

D. none of these


Answer:

We are given that,


We need to find the value of e.


We know that,


Determinant of 3 × 3 matrix is given as,




So,




⇒ (x2 + 3x)[-2x × 3x – (x – 4)(x + 4)] – (x – 1)[(x + 1) × 3x – (x – 4)(x – 3)] + (x + 3)[(x + 1)(x + 4) – (-2x)(x – 3)] = ax4 + bx3 + cx2 + dx + e


⇒ (x2 + 3x)[-6x – (x2 – 16)] – (x – 1)[3x(x + 1) – (x2 – 3x – 4x + 12)] + (x + 3)[x2 + x + 4x + 4 + 2x(x – 3)] = ax4 + bx3 + cx2 + dx + e


⇒ (x2 + 3x)[-6x – x2 + 16] – (x – 1)[3x2 + 3x – x2 + 7x – 12] + (x + 3)[x2 + 5x + 4 + 2x2 – 6x] = ax4 + bx3 + cx2 + dx + e


⇒ -x4 – 6x3 + 16x2 – 3x3 – 18x2 + 48x – (x – 1)[2x2 + 10x – 12] + (x + 3)[3x2 – x + 4] = ax4 + bx3 + cx2 + dx + e


⇒ -x4 – 9x3 – 2x2 + 48x – (2x3 – 2x2 + 10x2 – 10x – 12x + 12) + 3x3 + 9x2 – x2 – 3x + 4x + 12 = ax4 + bx3 + cx2 + dx + e


⇒ -x4 – 9x3 – 2x2 + 48x – 2x3 + 2x2 – 10x2 + 10x + 12x – 12 + 3x3 + 9x2 – x2 – 3x + 4x + 12 = ax4 + bx3 + cx2 + dx + e


⇒ -x4 – 9x3 – 2x3 + 3x3 – 2x2 + 2x2 + 9x2 – x2 + 48x + 10x + 12x – 3x + 4x – 12 + 12 = ax4 + bx3 + cx2 + dx + e


⇒ -x4 – 8x3 + 8x2 + 23x + 0 = ax4 + bx3 + cx2 + dx + e


Comparing left hand side and right-hand side of the equation, we get


e = 0


Thus, e = 0.


Question 10.

Mark the correct alternative in the following:

Using the factor theorem it is found that a + b, b + c and c + a are three factors of the determinant . The other factor in the value of the determinant is

A. 4

B. 2

C. a + b + c

D. none of these


Answer:


Let assume a=0, b=1, c=2




Now expending around colume1


0-1(-4-6)+2(3+4)=k(1)(3)(2)


6k=24


K=4


Question 11.

Mark the correct alternative in the following:

If a, b, c are distinct then the value of x satisfying is

A. c

B. a

C. b

D. 0


Answer:




(this is possible when x=0)


Question 12.

Mark the correct alternative in the following:

If the determinant , then

A. a, b, c are in H.P.

B. α is a root of 4ax2 + 12bx + 9c = 0 or, a, b, c are in G.P.

C. a, b, c are in G.P. only

D. a, b, c are in A.P.


Answer:

expend the determinats


a[-(2bα+3c)2 ]-b[-(2bα+3c)(2aα+3b)]+ (2aα+3b)[b(2bα+3c)-c(2aα+3b)]=0


-a(2bα+3c)2 + b(2bα+3c)(2aα+3b)+(2aα+3b)[2b^2 α+3bc-3bc-2acα]=0


(2bα+3c) [-2abα-3ac+2abα+3b2 ]+ (2aα+3b)(2α)( b2 -ac)=0


(2bα+3c) [-3ac +3b2 ]+ (2aα+3b)(2α)( b2-ac)=0


(b2 – ac)[4aα2 + 12bα + ac] = 0=


CASE1→(b2 -ac)=0


b2 =ac {abc are in Gp}


CASE2→(4aα2 +12bα+ac)=0 {Whose one root is }


Question 13.

Mark the correct alternative in the following:

If ω is a non-real cube root of unity and n is not a multiple of 3, then is equal to

A. 0

B. ω

C. ω2

D. 1


Answer:

Assume that n=2(not multiple of 3)



expend the determinant


∆=1(1-w3)-w2 (w-w4 )+w(w2 -w2 )


∆=1- w3- w3+w6 + w3- w3


∆=0


Question 14.

Mark the correct alternative in the following:

If , then the value of is

A. n

B. 2n

C. -2n

D. n2


Answer:

=

assume (n)=1



1(4-1)-1(8-1) +2(2-1)=-2


Answer=c(-2n)


Question 15.

Mark the correct alternative in the following:

If a > 0 and discriminant of ax2 + 2bx + c is negative, then is

A. positive

B. (ac – b2)(ax2 + 2bx + c)

C. negative

D. 0


Answer:

discriminant of ax2 + 2bx + c =0

and ax2 + 2bx + c



R3→ R3 – X R1 – R2



- (2ax + 2bx +c)(-b2 + ac) < 0


Question 16.

Mark the correct alternative in the following:

The value of is

A. 52

B. 0

C. 513

D. 59


Answer:




∆=0


Question 17.

Mark the correct alternative in the following:



A. 7

B. 10

C. 13

D. 17


Answer:

=








=10


Question 18.

Mark the correct alternative in the following:

If a, b, c are in A.P., then the determinant

A. 0

B. 1

C. x

D. 2x


Answer:


{a + c=2b}


R1→ R1 – R2


R2→ R2 – R3



R1→ R1 – R2



∆=0


Question 19.

Mark the correct alternative in the following:

If A + B + C = π, then the value of is equal to

A. 0

B. 1

C. 2sin B tan A cos C

D. none of these


Answer:






∆=0


Question 20.

Mark the correct alternative in the following:

The number of distinct real roots of lies in the interval is

A. 1

B. 2

C. 3

D. 0


Answer:





∆=(csc x+2 sec x )[(csc x-sec x)2 ]


Case1: (csc x+2 sec x )=0


(1st real root)


Case:(csc x-sec x)2 =0


Tan x=1 (2nd real root)


Question 21.

Mark the correct alternative in the following:

Let , where 0 ≤ θ ≤ 2π. Then,

A. Det (A) = 0

B. Det (A) ∈ (2, ∞)

C. Det (A) ∈ (2, 4)

D. Det (A) ∈ [2, 4]


Answer:

A=




A=2[(+1] 0≤(sin θ)2 ≤1


A∈2[1,2]


A∈[2,4]


Question 22.

Mark the correct alternative in the following:

If , then x =

A. 3

B. ± 3

C. ± 6

D. 6


Answer:


2x2 -40=18+14


x=±6


Question 23.

Mark the correct alternative in the following:

If , then

A. f(a) = 0

B. 3bc

C. a3 + b3 + c3 – 3abc

D. none of these


Answer:


ON TRANSPOSING







Question 24.

Mark the correct alternative in the following:

The value of the determinant is

A. a3 + b3 + c3

B. 3bc

C. a3 + b3 + c3 – 3abc

D. none of these


Answer:

assume a=1,b=2, c=3 (put in determinant)



∆=[-1(12-6)-5(3-4)+1(3-6)]


∆=-4


put a=1,b=2, c=3 in option A,B,C,D


ANSWER=D(none of these )


Question 25.

Mark the correct alternative in the following:

If x, y, z are different from zero and , then the value of is

A. xyz

B.

C. –x –y – z

D. –1


Answer:





x[(1+y)z+ y]-1[-yz]=0


xz + xyz+ xy+yz =0 (divide by xyz in both side)



Question 26.

Mark the correct alternative in the following:

The determinant equals

A. abc(b – c)(c – a)(a – b)

B. (b – c)(c – a)(a – b)

C. (a + b + c)(b – c)(c – a)(a – b)

D. none of these


Answer:

assume a=1,b=2, c=3 (put in determinant)





puta=1,b=2, c=3 in option A,B,C,D


ANSWER=D(none of these )


Question 27.

Mark the correct alternative in the following:

If x, y ∈ R, then the determinant lies in the interval

A. B. [–1, 1]

C. D.


Answer:


=






Δ = ( sin y – cos y)[(cos x )2 + ( sin x)2]


= ( sin y – cos y)


= - ( cos y – sin y)






Question 28.

Mark the correct alternative in the following:

The maximum value of is (θ is real)

A.

B.

C.

D.


Answer:




∆=cos θ (1-1-sin θ)


∆=-cos θ sin θ



-1 ≤sin 2θ≤1


[]


Question 29.

Mark the correct alternative in the following:

The value of the determinant is

A. 9x2(x + y)

B. 9y2(x + y)

C. 3y2(x + y)

D. 7x2(x + y)


Answer:











Question 30.

Mark the correct alternative in the following:

Let , then is equal to

A. 0

B. –1

C. 2

D. 3


Answer:




+




ANSWER= -1+1=0


Question 31.

Mark the correct alternative in the following:

There are two values of a which makes the determinant equal to 86. The sum of these two values is

A. 4

B. 5

C. –4

D. 9


Answer:




Sum of roots = [b=1and a=1]


Sum of roots= -4


Question 32.

Mark the correct alternative in the following:

If , t hen the value of is

A. 4

B. 8

C. 16

D. 32


Answer:




+



2×16=32


Question 33.

Mark the correct alternative in the following:

The value of is

A. 2

B. 4

C. 8

D. n2







Answer:






∆= 1/2 [8n+20-8n-4]


∆=8



Very Short Answer
Question 1.

If A is a singular matrix, then write the value of |A|.


Answer:

Since a singular matrix is a matrix whose determinant is 0, Therefore the determinant of A is 0.



Question 2.

Write the value of the determinant .


Answer:

Let

Using the property that if the equimultiples of corresponding elements of other rows (or columns) are added to every element of any row (or column) of a determinant, then the value of determinant remains the same


Using row transformation, R2→R2-xR1



Using the property that if all elements of any row or column of a determinant are 0, then the value of determinant is 0.


Since R2 has all elements 0, therefore ∆=0.



Question 3.

State whether the matrix is singular or non-singular.


Answer:

Let

Then


=2×4-3×6


=-10 (Expanding along R1)


Since |A|≠0, therefore A is a non-singular matrix.



Question 4.

Find the value of the determinant .


Answer:

Let


Using the property that if some or all elements of a row or column of a determinant are expressed as the sum of two (or more) terms, then the determinant can be expressed as the sum of two (or more) determinants.


We get,


Using the property that If any two rows (or columns) of a determinant are identical (all corresponding elements are same), then the value of the determinant is zero.


Hence,


=0×3-1×2=-2 (Expanding along R1)



Question 5.

Find the value of the determinant .


Answer:

Let


Using the property that if some or all elements of a row or column of a determinant are expressed as the sum of two (or more) terms, then the determinant can be expressed as the sum of two (or more) determinants.


We get,


Using the property that If any two rows (or columns) of a determinant are identical (all corresponding elements are same), then the value of the determinant is zero.


We get,



Using the property that if the equimultiples of corresponding elements of other rows (or columns) are added to every element of any row (or column) of a determinant, then the value of determinant remains the same.


Using row transformation, R2→R2-R1 and R3=R3-R1


We get,




Using the property that if each element of a row (or a column) of a determinant is multiplied by a constant k, then its value gets multiplied by k.


Taking out factor 2 from R3,


We get,


Using the property that If any two rows (or columns) of a determinant are identical (all corresponding elements are same), then the value of the determinant is zero.


Since, R2 and R3 are identical, therefore ∆ =0.



Question 6.

Write the value of the determinant .


Answer:

Let

Using the property that if the equimultiples of corresponding elements of other rows (or columns) are added to every element of any row (or column) of a determinant, then the value of determinant remains the same.


Using column transformation, C1→C1+C3


We get,


Using the property that if each element of a row (or a column) of a determinant is multiplied by a constant k, then its value gets multiplied by k.


Taking out factor(a+b+c) from C1,


We get,


Using column transformation, C1→C1-C2


We get,



Expanding along C1, we get


∆ =(a + b + c)×[(1-a)(c + a-(b + c))]=(1-a)(a-b)(a + b + c)



Question 7.

If and , find the value of |A| + |B|.


Answer:

Given that and , we have to find |A|+|B|

Then, and


|A|=0×1-i×i


=-i2


=1 (Expanding along R1 and since i2=-1)


|B|=0×1-1×1


=-1 (Expanding along R1)


|A|+|B|=1-1


0



Question 8.

If and , find |AB|.


Answer:

Given that and , we have to find |AB|

Then, and


|A|=1×-1-2×3


=-7 (Expanding along R1)


|B|=1×0-0×-1


=0 (Expanding along R1)


Since |AB|=|A||B|,


Therefore |AB|=-7×0=0



Question 9.

Evaluate: .


Answer:

Let

Using the property that if some or all elements of a row or column of a determinant are expressed as the sum of two (or more) terms, then the determinant can be expressed as the sum of two (or more) determinants.


We get,


Using the property that If any two rows (or columns) of a determinant are identical (all corresponding elements are same), then the value of the determinant is zero.


Hence,


=0×6-2×4=-8 (Expanding along R1)



Question 10.

If w is an imaginary cube root of unity, find the value of .


Answer:

Let

Using the property that if the equimultiples of corresponding elements of other rows (or columns) are added to every element of any row (or column) of a determinant, then the value of determinant remains the same


Using row transformation, R2→R2-ωR1



(Since, ω is a cube root of 1, therefore ω3=1)


Using the property that if all elements of a row or column of a determinant are 0, the value of determinant is 0.


Hence ∆=0



Question 11.

If and , find .


Answer:

Given that and , we have to find |AB|

Then, and


|A|=1×-1-2×3


=-7 (Expanding along R1)


|B|=1×(-2)-(-4)×3


=10 (Expanding along R1)


Since |AB|=|A||B|,


Therefore |AB|=-7×10=-70



Question 12.

If A = [aij] is a 3 × 3 diagonal matrix such that a11 = 1, a22 = 2 and a33 = 3, then find |A|.


Answer:

Since A is a diagonal matrix, therefore, all it’s non-diagonal members are 0. And a11=1, a22=2 and a33=3

We get


Then,


Expanding along R1


|A|= 1(2×3-0)=6



Question 13.

If A = [aij] is a 3 × 3 scalar matrix such that a11 = 2, then write the value of |A|.


Answer:

A scalar matrix is a matrix of order m which is equal to a constant λ multiplied with the Identity matrix of order m.

Since a11=2, hence λ=2 and m=3


Hence


Then,


Expanding along R1


|A|=2(2×2-0)


=8



Question 14.

If I3 denotes identity matrix of order 3 × 3, write the value of its determinant.


Answer:


Then,


Expanding along R1


|I3|=1(1×1-0)


=1



Question 15.

A matrix A of order 3 × 3 has determinant 5. What is the value of |3A|?


Answer:

If the determinant of a matrix A of order m is ∆, then the determinant of matrix λA, where λ is a scalar, is λm∆.

In this question, ∆=5, λ=3 and m=3.


|λA|=33×5


=135



Question 16.

On expanding by first row, the value of the determinant of 3 × 3 square matrix A = [aij] is a11C11 + a12C12 + a13C13, where Cij is the cofactor of aij is the cofactor of aij in A. Write the expression for its value of expanding by second column.


Answer:

The value of determinant written in the form of cofactors is equal to the sum of products of elements of that row (or column) multiplied by their corresponding cofactors.

Hence, the value of determinant |A|, of matrix A=[aij] of order 3×3, expanded along column 2 will be


|A|=a12×C12+a22×C22+a32×C32



Question 17.

Let A = [aij] be a square matrix of order 3 × 3 and Cij denote cofactor of aij in A. If |A| = 5, write the value of a31C31 + a32C32 + a33C33.


Answer:

The value of determinant |A||, of matrix A=[aij] of order 3×3, is given to be 5.

The value of determinant written in the form of cofactors is equal to the sum of products of elements of that row (or column) multiplied by their corresponding cofactors.


The value of |A| expanded along row 3 will be


|A|= a31×C31+a32×C32+a33×C33, which is the required expression


Hence, the value of required expression is equal to |A|=5.



Question 18.

In question 18, write the value of a11C21 + a12C22 + a13C23.


Answer:

We have to find out the value of a11×C21+a12×C22+a13×C23

LetI=a11×C21+a12×C22+a13×C23


I= a11×-(a12a33-a13a32)+a12×(a11a33-a13a31)+a13×-(a11a32-a12a31)


I=-a11a12a33+a11a13a32+a11a12a33-a12a13a31-a11a13a32+a12a13a31


I= a11a12a33–a11a12a33+a11a13a32-a11a13a32+ a12a13a31- a12a13a31


I=0



Question 19.

Write the value of .


Answer:

Let

Expanding along R1,


we get ∆= sin20°cos70°-(-cos20°)sin70°


= sin20°cos70°+cos20°sin70°


Since sin(A+B)= sinAcosB+cosAsinB


Hence, sin20°cos70°+cos20°sin70°=sin(20°+70°)


=sin(90°)


=1


Hence, ∆=1



Question 20.

If A is a square matrix satisfying ATA = I, write the value of |A|.


Answer:

Since ATA=I

Taking determinant both sides


|ATA|=|I|


Using |AB|=|A||B|,


|AT|=|A| and |I|=1, we get


|A||A|=1


(|A|)2=1


Hence, |A|=±1



Question 21.

If A and B are square matrices of the same order such that |A| = 3 and AB = I, then write the value of |B|.


Answer:

Given that |A|=3 and AB=I

Since AB=I


Taking determinant both sides


|AB|=|I|


Using |AB|=|A||B|,|A|=3 and |I|=1, we get


3|B|=1


Hence,



Question 22.

A is skew-symmetric of order 3, write the value of |A|.


Answer:

Since A is a skew-symmetric matrix, Therefore

AT=-A


Taking determinant both sides


|AT|=|-A|


Using |AT|=|A| and |λA|=λm|A| where m is the order of A


|A|=(-1)3|A|


=-|A| or 2|A|=0


Hence, |A|=0



Question 23.

If A is a square matrix of order 3 with determinant 4, then write the value of |-A|.


Answer:

Since |λA|= λm|A|

Given that λ=-1, m=3 and |A|=4, we get


|-A|=(-1)3×4=-4



Question 24.

If A is a square matrix such that |A| = 2, write the value of |AAT|.


Answer:

Given that |A|=2, we have to find |AAT|

Using |AB|=|A||B| and |AT|=|A|, we get


|AAT|=|A||AT|


=|A||A|


=2×2


=4



Question 25.

Find the value of the determinant .


Answer:

Let

Using the property that if the equimultiples of corresponding elements of other rows (or columns) are added to every element of any row (or column) of a determinant, then the value of determinant remains the same


Using row transformation, R1→R1-3R2


We get,



Using the property that if all elements of a row or column of a determinant are 0, the value of determinant is 0.


Hence ∆=0



Question 26.

Write the value of the determinant .


Answer:

Let


Using the property that if each element of a row (or a column) of a determinant is multiplied by a constant k, then its value gets multiplied by k.


Taking out factor 2 from R2 and 3 from R3,


We get,


Using the property that If any two rows (or columns) of a determinant are identical (all corresponding elements are same), then the value of the determinant is zero.


Since R1, R2 and R3 are identical, therefore ∆=0



Question 27.

If the matrix is singular, find the value of x.


Answer:

Let

Then,


=5x×1-2×-10 (Expanding along R1)


|A|=5x+20


For A to be singular, |A|=0


Hence 5x+20=0 or x=-4



Question 28.

If A is a square matrix of order n × n such that |A| = λ, then write the value of |-A|.


Answer:

Since |kA|= km|A|

Given that k=-1, m=n and |A|=λ, we get


|-A|=(-1)n×λ


Hence, |-A|=λ if n is even and |-A|=-λ if n is odd.



Question 29.

Find the value of the determinant .


Answer:

Let

Using the property that if each element of a row (or a column) of a determinant is multiplied by a constant k, then its value gets multiplied by k.


Taking out factor 22 from R1 and 23 from R2,



Using the property that If any two rows (or columns) of a determinant are identical (all corresponding elements are same), then the value of the determinant is zero.


Since R1 and R2 are identical, therefore ∆=0.



Question 30.

If A and B are non-singular matrices of the same order, write whether AB is singular or non-singular.


Answer:

We are given that,

A = non-singular matrix


B = non-singular matrix


Order of A = Order of B


We need to find whether AB is singular or non-singular.


Let us recall the definition of non-singular matrix.


Non-singular matrix, also called regular matrix, is a square matrix that is not singular, i.e., one that has a matrix inverse.


We can say that, a square matrix A is non-singular matrix iff its determinant is non-zero, i.e., |A| ≠ 0.


While a singular matrix is a square matrix that doesn’t have a matrix inverse. Also, the determinant is zero, i.e., |A| = 0.


So,


By definition, |A| ≠ 0 and |B| ≠ 0 since A and B are non-singular matrices.


Let,


Order of A = Order of B = n × n


⇒ Matrices A and B can be multiplied


⇒ A × B = AB


If we have matrices A and B of same order then we can say that,


|AB| = 0 iff either |A| or |B| = 0.


And it is clear that, |A|, |B| ≠ 0.


⇒ |AB| ≠ 0


And if |AB| ≠ 0, then by definition AB is s non-singular matrix.


Thus, AB is a singular matrix.



Question 31.

A matrix of order 3 × 3 has determinant 2. What is the value of |A(3I)|, where I is the identity matrix of order 3 × 3.


Answer:

We are given that,

Order of a matrix = 3 × 3


Determinant = 2


I = Identity matrix of order 3 × 3


We need to find the value of |A(3I)|.


Let the given matrix be A.


Then, |A| = 2


Also, since I is an identity matrix, then





⇒ Det (I) = 1


Or,


|I| = 1


Then, we can say


3(I) = 3


⇒ 3I = 3


Thus,


|A(3I)| = |A(3)| [∵, 3I = 3]


⇒ |A(3I)| = |3A|


By property of determinants, we know that


|KA| = Kn|A|, if A is of nth order.


⇒ |A(3I)| = 33|A| [∵, A has an order of 3 × 3 ⇒ |3A| = 33 |A|]


⇒ |A(3I)| = 27 |A|


Since, |A| = 2. Then,


⇒ |A(3I)| = 27 × 2


⇒ |A(3I)| = 54


Thus, |A(3I)| = 54.



Question 32.

If A and B are square matrices of order 3 such that |A| = -1, |B| = 3, then find the value of |3AB|.


Answer:

We are given that,

A and B are square matrices of order 3.


|A| = -1, |B| = 3


We need to find the value of |3AB|.


By property of determinant,


|KA| = Kn|A|


If A is of nth order.


If order of A = 3 × 3


And order of B = 3 × 3


⇒ Order of AB = 3 × 3 [∵, Number of columns in A = Number of rows in B]


We can write,


|3AB| = 33|AB| [∵, Order of AB = 3 × 3]


Now, |AB| = |A||B|.


⇒ |3AB| = 27|A||B|


Putting |A| = -1 and |B| = 3, we get


⇒ |3AB| = 27 × -1 × 3


⇒ |3AB| = -81


Thus, the value of |3AB| = -81.



Question 33.

Write the value of .


Answer:

We need to find the value of


Determinant of 2 × 2 matrix is found as,



So,



Rearranging,



Using the algebraic identity,


(x + y)(x – y) = x2 – y2





Here, i is iota, an imaginary number.


Note that,


i2 = -1


So,




Thus,




Question 34.

Write the cofactor of a12 in the matrix .


Answer:

We need to find the cofactor of a12 in the matrix


A cofactor is the number you get when you remove the column and row of a designated element in a matrix, which is just a numerical grid in the form of a rectangle or a square. The cofactor is always preceded by a positive (+) or negative (-) sign, depending whether the element is in a + or - position. It is



Let us recall how to find the cofactor of any element:


If we are given with,



Cofactor of any element, say a11 is found by eliminating first row and first column.



⇒ Cofactor of a11 = a22 × a33 – a23 × a32


The sign of cofactor of a11 is (+).


And, cofactor of any element, say a12 is found by eliminating first row and second column.



⇒ Cofactor of a12 = a21 × a33 – a23 × a31


The sign of cofactor of a12 is (-).


Similarly,


First know what the element at position a12 in the matrix is.


In ,


a12 = -3


And as discussed above, the sign at a12 is (-).


For cofactor of -3, eliminate first row and second column in the matrix.



⇒ Cofactor of -3 = (6 × -7) – (4 × 1)


⇒ Cofactor of -3 = -42 – 4


⇒ Cofactor of -3 = -46


Since, the sign of cofactor of -3 is (-), then


Cofactor of -3 = -(-46)


⇒ Cofactor of -3 = 46


Thus, cofactor of -3 is 46.



Question 35.

If find x.


Answer:

9(2x + 5) – 3(5x+2) = 0


⇒ 18x + 45 – 15x – 6 = 0


⇒ 3x + 39 = 0


⇒ 3x = - 39


⇒ x = -13



Question 36.

Find the value of x from the following: .


Answer:

We are given that,


We need to find the value of x.


Determinant of 2 × 2 matrix is found as,



So, determinant of the given matrix is found as,




According to the question, equate this to 0.


2x2 – 4 = 0


We need to solve the algebraic equation.


2x2= 4



⇒ x2 = 2


Taking square root on both sides of the equation,



⇒ x = ±√2


Hence, the value of x is ±√2.



Question 37.

Write the value of the determinant .


Answer:

We need to find the value of determinant,


Determinant of 3 × 3 matrices is found as,




Similarly,








Thus, the value of .



Question 38.

If |A| = 2, where A is 2 × 2 matrix, find |adj A|.


Answer:

We are given that,

Order of matrix A = 2 × 2


|A| = 2


We need to find the |adj A|.


Let us understand what adjoint of a matrix is.


Let A = [aij] be a square matrix of order n × n. Then, the adjoint of the matrix A is transpose of the cofactor of matrix A.


The relationship between adjoint of matrix and determinant of matrix is given as,


|adj A| = |A|n-1


Where, n = order of the matrix


Putting |A| = 2 in the above equation,


⇒ |adj A| = (2)n-1 …(i)


Here, order of matrix A = 2


∴, n = 2


Putting n = 2 in equation (i), we get


⇒ |adj A| = (2)2-1


⇒ |adj A| = (2)1


⇒ |adj A| = 2


Thus, the |adj A| is 2.



Question 39.

For what is the value of the determinant ?


Answer:

We need to find the value of determinant,


Determinant of 3 × 3 matrices is found as,




Similarly,








Thus, the value of is 8.



Question 40.

For what value of x is the matrix singular?


Answer:

We are given that,

is singular matrix.


We need to find the value of x.


Let us recall the definition of singular matrix.


A singular matrix is a square matrix that doesn’t have a matrix inverse. A matrix ‘A’ is singular iff its determinant is zero, i.e., |A| = 0.


Hence, we just need to find the determinant of the given matrix and equate it to zero.


Determinant of 2 × 2 matrix is found as,



So,







Now, equate this to 0.


That is,



⇒ 3x – 6 = 0


⇒ 3x = 6



⇒ x = 2


Thus, the value of x = 2 for which the matrix is singular.



Question 41.

A matrix A of order 3 × 3 is such that |A| = 4. Find the value of |2A|.


Answer:

We are given that,

Order of matrix A = 3


|A| = 4


We need to find the value of |2A|.


By property of determinant of matrix,


|KA| = Kn|A|


Where, order of the matrix A is n.


Similarly,


|2A| = 23|A|


[∵, Order of matrix A = 3]


⇒ |2A| = 8|A|


Substituting the value of |A| in the above equation,


⇒ |2A| = 8 × 4


⇒ |2A| = 32


Thus, the value of |2A| is 32.



Question 42.

Evaluate:


Answer:

We need to evaluate the matrix:


Determinant of 2 × 2 matrix is found as,



So,



Using the trigonometric identity,


cos (A + B) = cos A cos B – sin A sin B


Replace A by 15° and B by 75°.


cos (15° + 75°) = cos 15° cos 75° - sin 15° cos 75°


⇒ cos 90° = cos 15° cos 75° - sin 15° cos 75°


So, substituting it, we get



Using the trigonometric identity,


cos 90° = 0



Thus, the value of .



Question 43.

If . Write the cofactor of the element a32.


Answer:

We are given that,


We need to find the cofactor of the element a32.


A cofactor is the number you get when you remove the column and row of a designated element in a matrix, which is just a numerical grid in the form of a rectangle or a square. The cofactor is always preceded by a positive (+) or negative (-) sign, depending whether the element is in a + or - position. It is



Let us recall how to find the cofactor of any element:


If we are given with,



Cofactor of any element, say a11 is found by eliminating first row and first column.



⇒ Cofactor of a11 = a22 × a33 – a23 × a32


The sign of cofactor of a11 is (+).


And, cofactor of any element, say a12 is found by eliminating first row and second column.



⇒ Cofactor of a12 = a21 × a33 – a23 × a31


The sign of cofactor of a12 is (-).


So,


In matrix, .


Element at a32 = 2


We need to find the cofactor of 2 at a32.


And as discussed above, the sign at a32 is (-).


For cofactor of a32, eliminate third row and second column in the matrix.



⇒ Cofactor of a32 = 5 × 1 – 8 × 2


⇒ Cofactor of a32 = 5 – 16


⇒ Cofactor of a32 = -11


Since, the sign of cofactor of a32 is (-), then


Cofactor of a32 = -(-11)


⇒ Cofactor of a32 = 11


Thus, cofactor of a32 is 11.



Question 44.

If , then write the value of x.


Answer:

We are given that,


We need to find the value of x.


Determinant of 2 × 2 matrix is found as,



Let us take left hand side (LHS) of the given matrix equation.



⇒ LHS = (x + 1)(x + 2) – (x – 1)(x – 3)


⇒ LHS = (x2 + x + 2x + 2) – (x2 – x – 3x + 3)


⇒ LHS = (x2 + 3x + 2) – (x2 – 4x + 3)


⇒ LHS = x2 + 3x + 2 – x2 + 4x – 3


⇒ LHS = x2 – x2 + 3x + 4x + 2 – 3


⇒ LHS = 7x – 1


Let us take right hand side (RHS) of the given matrix equation.



⇒ RHS = 4 × 3 – (-1) × 1


⇒ RHS = 12 + 1


⇒ RHS = 13


Now,


LHS = RHS


⇒ 7x – 1 = 13


⇒ 7x = 13 + 1


⇒ 7x = 14



⇒ x = 2


Thus, the value of x is 2.



Question 45.

If , then write the value of x.


Answer:

We are given that,


We need to find the value of x.


Determinant of 2 × 2 matrix is found as,



Let us take left hand side (LHS) of the given matrix equation.



⇒ LHS = 2x(x + 1) – (x + 3)(2(x + 1))


⇒ LHS = (2x2 + 2x) – (x + 3)(2x + 2)


⇒ LHS = (2x2 + 2x) – (2x2 + 2x + 6x + 6)


⇒ LHS = (2x2 + 2x) – (2x2 + 8x + 6)


⇒ LHS = 2x2 + 2x – 2x2 – 8x – 6


⇒ LHS = 2x2 – 2x2 + 2x – 8x – 6


⇒ LHS = -6x – 6


Let us take right hand side (RHS) of the given matrix equation.



⇒ RHS = 1 × 3 – 5 × 3


⇒ RHS = 3 – 15


⇒ RHS = -12


Now,


LHS = RHS


⇒ -6x – 6 = -12


⇒ -6x = -12 + 6


⇒ -6x = -6



⇒ x = 1


Thus, the value of x is 1.



Question 46.

If , find the value of x.


Answer:

We are given that,


We need to find the value of x.


Determinant of 2 × 2 matrix is found as,



Let us take left hand side (LHS) of the given matrix equation.



⇒ LHS = 3x × 4 – 7 × (-2)


⇒ LHS = 12x – (-14)


⇒ LHS = 12x + 14


Let us take right hand side (RHS) of the given matrix equation.



⇒ RHS = 8 × 4 – 7 × 6


⇒ RHS = 32 – 42


⇒ RHS = -10


Now,


LHS = RHS


⇒ 12x + 14 = -10


⇒ 12x = -10 – 14


⇒ 12x = -24



⇒ x = -2


Thus, the value of x is -2.



Question 47.

If , write the value of x.


Answer:

We are given that,


We need to find the value of x.


Determinant of 2 × 2 matrix is found as,



Let us take left hand side (LHS) of the given matrix equation.



⇒ LHS = 2x × x – 5 × 8


⇒ LHS = 2x2 – 40


Let us take right hand side (RHS) of the given matrix equation.



⇒ RHS = 6 × 3 – (-2) × 7


⇒ RHS = 18 – (-14)


⇒ RHS = 18 + 14


⇒ RHS = 32


Now,


LHS = RHS


⇒ 2x2 – 40 = 32


⇒ 2x2 = 32 + 40


⇒ 2x2 = 72



⇒ x2 = 36


⇒ x = ±√36


⇒ x = ±6


Thus, the value of x is ±6.



Question 48.

If A is a 3 × 3 matrix, |A| ≠ 0 and |3A| = k|A| then write value of k.


Answer:

We are given that,

Order of matrix = 3


|A| ≠ 0


|3A| = k|A|


We need to find the value of k.


In order to find k, we need to solve |3A|.


Using property of determinants,


|kA| = kn|A|


Where, order of A is n × n.


Similarly,


|3A| = 33|A|


[∵, order of A is 3]


⇒ |3A| = 27|A| …(i)


As, according to the question


|3A| = k|A|


Using (i),


⇒ 27|A| = k|A|


Comparing the left hand side and right hand side, we get


k = 27


Thus, the value of k is 27.



Question 49.

Write the value of the determinant .


Answer:

We need to find the determinant,


Determinant of 2 × 2 matrix is found as,



So,



Using the algebraic identity,


(a + b)(a – b) = a2 – b2





Thus, the value of .



Question 50.

Write the value of the determinant .


Answer:

We need to find the value of determinant


Determinant of 3 × 3 matrices is found as,




So,






Re-arranging the equation,




Using the algebraic identity,


(a + b)(a – b) = a2 – b2







Thus, the value of is 0.



Question 51.

If , then for any natural number, find the value of Det(An).


Answer:

We are given that,


We need to find the det(An).


To find det(An),


First we need to find An, and then take determinant of An.


Let us find A2.


A2 = A.A



Let,



For z11: Dot multiply the first row of the first matrix and first column of the second matrix, then sum up.


That is,


(cos θ, sin θ).(cos θ, -sin θ) = cos θ × cos θ + sin θ × (-sin θ)


⇒ (cos θ, sin θ).(cos θ, -sin θ) = cos2 θ – sin2 θ


By algebraic identity,


cos 2θ = cos2 θ – sin2 θ


⇒ (cos θ, sin θ).(cos θ, -sin θ) = cos 2θ



For z12: Dot multiply the first row of the first matrix and second column of the second matrix, then sum up.


That is,


(cos θ, sin θ)(sin θ, cos θ) = cos θ × sin θ + sin θ × cos θ


⇒ (cos θ, sin θ)(sin θ, cos θ) = sin θ cos θ + sin θ cos θ


⇒ (cos θ, sin θ)(sin θ, cos θ) = 2 sin θ cos θ


By algebraic identity,


sin 2θ = 2 sin θ cos θ


⇒ (cos θ, sin θ)(sin θ, cos θ) = sin 2θ



Similarly,







If and , then



Now, taking determinant of An,



Determinant of 2 × 2 matrix is found as,



So,


Det(An) = cos nθ × cos nθ – sin nθ × (-sin nθ)


⇒ Det(An) = cos2 nθ + sin2


Using the algebraic identity,


sin2 A + cos2 A = 1


⇒ Det(An) = 1


Thus, Det(An) is 1.



Question 52.

Find the maximum value of .


Answer:

We need to find the maximum value of


Let us find the determinant,



Determinant of 3 × 3 matrices is found as,




So,








Multiply and divide by 2 on right hand side,




[∵, By trigonometric identity, sin 2θ = 2 sin θ cos θ]


We need to find the maximum value of .


We know the range of sine function.


-1 ≤ sin A ≤ 1


Or,


-1 ≤ sin 2θ ≤ 1


∴, maximum value of sin 2θ is 1.


⇒ maximum value of = 1/2


Thus, maximum value of



Question 53.

If x ∈ N and , then find the value of x.


Answer:

We are given that,


x ∈ N


We need to find the value of x.


Determinant of 2 × 2 matrix is found as,



So, take





Since,



⇒ 2x2 = 8



⇒ x2 = 4


⇒ x = ±√4


⇒ x = ±2


Since, x ∈ N


-2 is not a natural number.


Thus, the value of x is 2.



Question 54.

If , write the value of x.


Answer:

We are given that,


We need to find the value of x.


Determinant of 3 × 3 matrices is found as,




So,








Using trigonometric identity,


sin2 θ + cos2 θ = 1




Since,



⇒ -x3 = 8


⇒ x3 = -8


⇒ x3 = -2 × -2 × -2


Taking cube root on both sides,



⇒ x = -2


Thus, the value of x is -2.



Question 55.

If A is a 3 × 3 matrix, then what will be the value of k if Det(A-1) = (Det A)k?


Answer:

We are given that,

Order of matrix = 3 × 3


Det(A-1) = (Det A)k


An n-by-n square matrix A is called invertible if there exists an n-by-n square matrix B such that where In denotes the n-by-n identity matrix and the multiplication used is ordinary matrix multiplication.


We know that,


If A and B are square matrices of same order, then


Det (AB) = Det (A).Det (B)


Since, A is an invertible matrix, this means that, A has an inverse called A-1.


Then, if A and A-1 are inverse matrices, then


Det (AA-1) = Det (A).Det (A-1)


By property of inverse matrices,


AA-1 = I


∴, Det (I) = Det (A).Det (A-1)


Since, Det (I) = 1


⇒ 1 = Det (A).Det (A-1)



⇒ Det (A-1) = Det (A)-1


Since, according to question,


Det(A-1) = (Det A)k


⇒ k = -1


Thus, the value of k is -1.